THE 66th ANNUAL ZOLA COOPER LEE T. NESBITT SEMINAR

A Clinical and Dermatopathologic Seminar

PRESENTED IN MEMORY OF ITS FOUNDER James W. Burks, III, M.D.

Also Honoring Long-time Contributors:

Marvin E. Chernosky, M.D. Herbert B. Christianson, M.D. Wallace H. Clark, M.D. Robert M. Fine, M.D. Robert G. Freeman, M.D. Lee T. Nesbitt, Jr. M.D. Charles Perniciaro, M.D. Richard J. Reed, M.D. Morris Waisman, M.D.

2019 Guest Dermatopathologist

Soon Bahrami, M.D. Associate Professor of and Director of Dermatopathology University of Louisville Louisville, Kentucky

New Orleans, LA

Saturday, November 2, 2019

1

THE 66th ANNUAL ZOLA COOPER LEE T. NESBITT SEMINAR

November 2, 2019 New Orleans, LA

PROGRAM

Panelists: Soon Bahrami, M.D. Tom Davis, M.D. Clay J. Cockerell, M.D. Mark Cappel, MD

Moderators: Sandra Osswald, M.D. Travis Vandergriff, M.D.

Agenda:

8:00– 9:15 a.m. Case presentations

9:15 – 9:45 a.m. Vincent Derbes Memorial Lecture Soon Bahrami, M.D. Skinternal Medicine: Selected cases illustrating cutaneous manifestations of internal diseases

9:45 – 10:15 a.m. Break

10:15 -11:55 a.m. Case presentations

11:55 a.m. Award presentations

12:00 p.m. Adjourn

2

THE 66th ANNUAL ZOLA COOPER LEE T. NESBITT SEMINAR

2019

OFFICERS AND FACULTY

President: Pamela Martin, M.D.

Director: Clay J. Cockerell, M.D.

Secretary/Treasurer: Travis Vandergriff, M.D.

Webmaster: Annie Morrison, M.D.

Panelists: Soon Bahrami, M.D. Tom Davis, M.D. Clay J. Cockerell, M.D. Mark Cappel, MD

Moderators: Sandra Osswald, M.D. Travis Vandergriff, M.D.

Board of Directors: Emeritus Members: Clay J. Cockerell M.D., Dallas, TX Marvin E. Chernosky M.D., Houston, TX Mark Cappel M.D., Tampa, FL Robert M. Fine M.D., Atlanta, GA Carlos Ricotti M.D., Miami, FL Robert G. Freeman M.D., Dallas, TX Scott Dalton D.O., San Antonio, TX Steven J. Hodge M.D., Louisville, KY Thomas Nicotri M.D., New Orleans, LA Jack Lesher M.D., Augusta, GA Travis Vandergriff M.D., Dallas, TX Emily Omura M.D., Birmingham, AL Erin Boh M.D., New Orleans, LA Charles V. Perniciaro M.D., Jacksonville, FL Tom Davis M.D., San Antonio, TX Ron Rapini M.D., Houston, TX Sandra Osswald M.D., San Antonio, TX Marilyn Ray M.D., New Orleans, LA John Millns M.D., Tampa, FL Richard Reed M.D., New Orleans, LA Pamela Martin M.D., New Orleans, LA Omar Sangueza M.D., Winston-Salem, NC Dipti Anand M.D., Atlanta, GA Leonard Gately M.D., Metairie, LA Jason Smith, M.D., Rome, GA Ted Rosen M.D., Houston, TX Kenneth Greer M.D., Houston, TX Thomas R. Wade M.D., Atlanta, GA Dirk Elston M.D., Charleston, SC L. Frank Glass M.D., Tampa, FL Jacqueline Sasaski M.D., Gulfport, MS

Visit our website at: www.zolacooper.org 3

HISTORICAL BACKGROUND

As Dr. Jim Burks relates in the 1956 program, it was suggested at the 1953 meeting of the American Academy of that a group of Southern dermatologists consider organizing a dermatopathologic seminar with the Southern Medical Association, similar to the seminar held with the meeting of the Pacific Dermatologic Association. Dr. Burks became the leader of this effort to organize such a seminar, one of the first ever established in this specialty as a continuing education program, but he credited Dr. Walter Nickel as an inspiration. In introducing Dr. Nickel as the guest dermatopathologist in 1958, Dr. Burks stated of Dr. Nickel “the creator and director of the CPC of the PDA for 10 years is largely responsible for the creation of this, the Zola Cooper Seminar.”

The 1954 meeting of the Southern Medical Association was scheduled in St. Louis. Dr. Burks sought the help of Zola K. Cooper, Ph.D., Assistant Professor of and Assistant Professor of Medicine (Dermatology) at Washington University School of Medicine in St. Louis, and Consultant Pathologist to the Barnard Free Skin and Cancer Hospital. Dr. Cooper had also served on the faculty of the University of Oklahoma School of Medicine from 1946 - 1949. Though she was not a , Dr. Cooper had become an expert in the knowledge and diagnosis of skin diseases. She was one of the foremost authorities in dermatopathology. She devoted much of her time in 1953 and 1954 to the planned Seminar, and was to have served as the first guest moderator. She was found dead in her room on October 23, 1954. The Seminar was subsequently named the Zola Cooper Seminar in her honor. With Dr. Cooper’s unexpected death, Dr. Hamilton Montgomery was selected as the guest dermatopathologist for the initial meeting in 1954. Others panelists participating in that 1954 meeting included Dr. Francis Lynch, the guest dermatologist of the Southern Medical Association’s Section on Dermatology; Dr. John H. Lamb of Oklahoma City, Oklahoma; Dr. Francis A. Ellis of Baltimore, Maryland; Dr. Morris Waisman of Tampa, Florida; Dr. Joseph M. Hitch of Raleigh, North Carolina; and Dr. Ed Cawley of Charlottesville, Virginia.

Dr. James W. Burks, III, eminent dermatologist, pioneer dermatologic surgeon, member of the faculty of Tulane University School of Medicine, teacher, friend of many, and devoted deep-sea fisherman, was the founder, and then the Director and Moderator of the Zola Cooper Seminar for the remainder of his life. He died after fighting a blue marlin on a deep- sea expedition on July 27, 1978. Jim had many interests, but he remained dedicated to the Zola Cooper Seminar. He co-moderated every meeting, remained firmly but gently in control as moderator, and always finished the discussion of cases on time. He was generous in giving credit to others, careful to enroll the aid of worthy contributors, and dedicated to maintaining the goal of the Seminar as a practical continuing educational exercise for the practicing physician. His associates, Dr. John Yarborough, Dr. Gary Brown, and Dr. George A. Farber contributed significantly as well. Since its inception, virtually every prominent name in American dermatopathology has participated at least once in this Seminar. After Dr. Burks’ death, the name of the Seminar was revised to the “James W. Burks-Zola Cooper Memorial Clinicopathologic Seminar” under the direction of George A. Farber, M.D.

In 1984, the Seminar was reorganized, continuing the affiliation with the Southern Medical Association, and was presented for ten years as the “Clinical and Dermatopathologic 4

Seminar.” In 1994, by general agreement, the memory and pioneering work of Dr. Zola Cooper were again recognized by renaming the Seminar as the “Zola Cooper Seminar presented in memory of its founder, James W. Burks, III, M.D.” Also honored are several other long-time contributors to the Seminar who have each served significantly in continuing the Burks tradition, many of whom participated for 35 or more years. Dr. Herbert Christianson was a Co-Director and Moderator from 1962 until his death; Dr. Wallace Clark contributed as a key dermatopathology panel member and “Faculty Moderator” at a critical time in the development of the Seminar from 1958 - 1973; Dr. Robert G. Freeman began participating in the Seminar in 1959, and he was the President and Director from 1984 - 1994; Dr. Richard J. Reed was been panel member, host, moderator, and significant contributor beginning in 1963; Dr. Morris Waisman was a panelist and Director of the Seminar from its inception in 1954, and was involved until his death; Dr. Marvin E. Chernosky was a long-time contributor, Director, Secretary-Treasurer and moderator from 1984 - 1995; Dr. Robert Fine was a generous contributor of cases and knowledge for many years; Dr. Lee T. Nesbitt, Chairman of the Dermatology at L.S.U. School of Medicine in New Orleans, was added as a recognized contributor in 2010. Dr. Nesbitt was an outstanding clinical dermatologist, and moderated and contributed cases to the Zola Cooper Seminar for over four decades, before his sudden and untimely death in 2014. The Zola Cooper Seminar Board of Directors voted in 2014 to rename the Seminar the “Zola Cooper-Lee T. Nesbitt Seminar,” as a tribute to Dr. Nesbitt’s lifelong support of the Seminar.

The 48th Zola Cooper Seminar was re-presented in Maui, Hawaii, and the 49th, 50th, and 51st Annual Seminars were presented in conjunction with the Fall meeting of the Louisiana Dermatological Society in New Orleans, Louisiana. The 52nd, 53rd, and 54th seminars were again held in conjunction with the Southern Medical Association in San Antonio, Texas, Charlotte, North Carolina, and New Orleans, Louisiana, respectively. The 55th was held in New Orleans with the combined meeting of Louisiana and Texas Dermatological Societies. The 56th Zola Seminar was held in association with the Texas Dermatology Society (Albuquerque, NM). In 2010, largely through the efforts of Dr. Nesbitt, the 57th Zola Cooper Seminar began a more permanent affiliation with the Louisiana Dermatological Society’s Fall meeting. In 2018, the Seminar was held independently in New Orleans.

Purpose:

The Zola Cooper-Lee T. Nesbitt Clinical and Dermatopathologic Seminar is an educational organization designed to promote continuous excellence in the field of clinical dermatology and dermatopathology.

Contributions:

Tax exempt contributions are welcome. Contributions can be made payable to:

Zola Cooper Seminar c/o Travis Vandergriff, M.D., Secretary-Treasurer 5730 Kenwood Ave Dallas, Texas 75206

The Seminar’s Federal Tax Identification Number is 30-0232262.

5

PAST CHAIRMEN, OFFICERS, MODERATORS AND PANELISTS

Chairmen: James W. Burks, III, M.D., Founder and Chairman, 1953 - 1978 Herb Christianson, M.D., Co-Director, 1962 - 1983 George A. Farber, Sr., M.D., Co-Director, 1974-1983 Gary R. Brown, M.D., Co-Director, 1974 Robert G. Freeman, M.D., President and Director, 1984 - 1994 Thomas R. Wade, M.D., Director, 1994 - 1997 Clay J. Cockerell, M.D., Director, 1998 - present

Secretary-Treasurers: James W. Burks, III, M.D., 1953 - 1966 John Yarborough, M.D., 1966 - 1969 George A. Farber, Sr., M.D., 1970 - 1983 Marvin E. Chernosky, M.D., 1984 - 1994 Lee T. Nesbitt, Jr., M.D., 1995 - 1997 Charles V. Perniciaro, M.D., 1998 – 2003 Marilyn Ray, M.D., 2003 – 2008 Tom Davis, M.D., 2009 – 2015 Travis Vandergriff, 2015 - present

Moderators: James W. Burks, III, M.D., 1954 –1978 Herbert B. Christianson, M.D., 1962 – 1993 George A. Farber, Sr., M.D., 1973 –1983 Marvin E. Chernosky, M.D., 1984 –1998 Lee T. Nesbitt, Jr., M.D., 1984, 1988, 1991 – 2013 Kenneth E. Greer, M.D., 1994 –1998 Emily F. Omura, M.D., 1996 – 2000 Jack Lesher, M.D., 1999 – 2000 Marilyn Ray, M.D., 2001 – 2008 Ted Rosen, M.D., 2001 – 2003, 2005 – 2008 Leonard Gately, 2003 – 2007, 2012 Clay Cockerell, M.D., 2009 Tom Davis, M.D., 2009 Jacqueline Sasaki, M.D., 2010, 2011, 2012 Tom Nicotri, M.D., 2010 Sandra Osswald, M.D., 2011 – present Travis Vandergriff, M.D., 2013 - present

Dermatopathology Panelists: Robert G. Freeman, M.D., 1959 – 1996 Richard J. Reed, M.D., 1960 – 1996; 2002 Omar Sangueza, M.D., 2002 - 2003 Charles V. Perniciaro, M.D., 2002 – 2017 Clay J. Cockerell, M.D., 1992 – present Thomas L. Davis, M.D., 2004 - 2005, 2007 – present Mark Cappel, M.D., 2013 - present

6

Administrative Secretaries: Marilyn Brown, 1971-1983 Julia Brinkman, 1984-1990 Pam Vitez 2002-2006 Tami Golden 2007 Janet Love 2010-2012 Tami Golden 2013 Brenda Wander 2014, 2015 Sondra Cockerell 2017-present

PAST HONORED GUEST DERMATOPATHOLOGISTS

1954 Hamilton Montgomery, M.D. 1986 Richard J. Reed, M.D. 1955 Marcus Caro, M.D. 1987 Antoinette Hood, M.D. 1956 Herman Beerman, M.D. & Evan Farmer, M.D. 1957 J. Walter Wilson, M.D. 1988 James H. Graham, M.D. 1958 Walter R. Nickel, M.D. 1989 N. Scott McNutt, M.D. 1959 Walter F. Lever, M.D. 1990 W. P. Daniel Su, M.D. 1960 Hamilton Montgomery, M.D. 1991 Kurt Stenn, M.D. 1961 Richard Stoughton, M.D. 1992 Clifton R. White, M.D. 1962 Frederick J. Szymanski, M.D. 1993 David N. Silvers, M.D. 1963 Richard K. Winkelman, M.D. 1994 Philip LeBoit, M.D. 1964 Robert W. Goltz, M.D. 1995 Daniel J. Santa Cruz, M.D. 1965 Herman Beerman, M.D. 1996 Hideko Kamino, M.D. 1966 Elson B. Helwig, M.D. 1997 Raymond L. Barnhill, M.D. 1967 Hermann Pinkus, M.D. 1998 Neal S. Penneys, M.D. 1968 James H. Graham, M.D. 1999 George P. Lupton, M.D. 1969 Arthur Hyman, M.D. 2000 James E. Fitzpatrick, M.D. 1970 Louis Winer, M.D. 2001 Alan S. Boyd, M.D. 1971 Herbert Z. Lund, M.D. 2002 Lawrence (Larry) Gibson, M.D. 1972 Daniel Richfield, M.D. 2003 Thomas D. Horn, M.D. 1973 Lewis Shapiro, M.D. 2004 Dirk Elston, M.D. 1974 Waine C. Johnson, M.D. 2005 Joan Guitart, M.D. 1975 Robert W. Goltz, M.D. 2006 John C. Maize, Sr., M.D. 1976 Walter R. Nickel, M.D. 2007 Anita Gilliam, M.D. 1977 Edward Wilson-Jones, M.D. 2008 Iftikhar Ahmed, M.D. 1978 A. Bernard Ackerman, M.D. 2009 Mark Jacobson, M.D. 1979 John T. Headington, M.D. 2010 Beth Ruben, M.D. 1980 Loren E. Golitz, M.D. 2011 Mary S. Stone, M.D. 1981 Amir H. Mehregan, M.D. 2012 Lori Lowe, M.D 1982 Martin Brownstein, M.D. 2013 Adam Rubin, M.D. 1983 Wilma Bergfeld, M.D. 2014 Martin C. Mihm, Jr., M.D. 1984 Ronald J. Barr, M.D. 2015 Margot Peters, M.D. 1985 William A. Caro, M.D. 2016 Pedram Gerami, M.D. 2017 Shawn Cowper, M.D. 2018 Tammie Ferringer, M.D.

7

Table of Contents

Case No. Diagnosis

1. Doubly Recurrent Nevus Mark A. Cappel, MD Ellenor Swartz, PA-C

2. Atypical Melanocytic Proliferation Julia Accetta, MD Lauren Chen, BS Andrea Murina, MD

3. Melanoma In Situ Histologically Simulating Terry Henges, PA-C “Dysplastic” Nevus Clay J. Cockerell, MD

4. Desmoplastic Melanoma Jason L. Smith, MD

5. Melanoma Masquerading as a Dermatofibroma Faryal Siddiqui, MD Francis Sharkey, MD Whitney High, MD Sandra Osswald, MD Daniel Glade, BS

6a. Linear Unilateral Basaloid Follicular Proliferation Mark A. Cappel, MD Kyle Bartlett, MD

6b. Basal Cell Nevus Syndrome in a 9 month Infant Jeffery Scott Harbold Jr, DO

6c. Basal Cell Carcinoma Transformation in a Nevus Sebaceus Lucy Chao, MD Abida Kadi, MD Leah Jacob, MD

7a. Atypical Fibroxanthoma Jason L. Smith, MD

7b. Pleomorphic Dermal Sarcoma Hope Retif, MS-II Taylor Smith, MD Keith Harris, MD

7c. Pleomorphic Dermal Sarcoma John Adnot, MD Wade Smith, MD Clay J. Cockerell, MD

8a. Sarcomatoid Squamous Cell Carcinoma Jason L. Smith, MD

8

8b. Cutaneous Squamo-Melanocytic Tumor Jason L. Smith, MD Keith R. Harris, MD

9a. Lamotrigine-Induced Cutaneous CD 30+ Anaplastic Large Dipti Anand, MD Cell Lymphoma Aditya Sood, M1 Caitlin Clealand, PA-C Abha Soni, DO

9b. Localized CD30 and CD4 Positive Cutaneous T-Cell Gage Rensch, MD Lymphona

10a. CD30+ Large Cell Anaplastic Lymphoma Daniel Witheiler, MD Clay J. Cockerell, MD

10b. CD30 Positive Lymphoma Simulating Pyoderma Leisa Hodges, DO Gangrenosum Clay J. Cockerell, MD

11. Metastatic Renal Cell Carcinoma Etan Marks, DO Clay J. Cockerell, MD

12. Neuroendocrine Carcinoma Helen Kaporis, DO Clay J. Cockerell, MD

13a. Primary Cutaneous Marginal Zone B-Cell Lymphoma Katharine Saussy, MD vs. Pseudolymphoma Andrea Murina, MD

13b. Cutaneous T-Cell Lymphoma and Concurrent Primary Garrett Vick, MD Cutaneous Marginal Zone B- Cell Lymphoma Lucy Chao, MD Abida Kadi, MD

14. Rare Diffuse Large B-Cell Lymphoma, Intravascular Type Lindsey Bush Derrick Moody, MD Dipti Anand, MD

15. Atypical Lymphocytic Lobular Panniculitis Aditya Sood,M1 Dustin Portela, DO Dipti Anand, MD

16. Steatocystoma Multiplex and Neurofibromatosis Adeline Kikam

17. Dermal Fibromatous Hamartoma Most Consistent with Leisa Hodges, DO Medallion-Like Dermal Dendrocytic Hamartoma Clay J. Cockerell, MD

9

18. Degos Disease Aditya Sood, M1 David Schoenfeld, MD Dipti Anand, MD

19. Dermatomyositis, most consistant with MDA-5 Variant Natalie Wright, MD Elba Rubianes, MD Clay J. Cockerell, MD

20. Cutaneous Collagenous Vasculopathy Ravi R. Patel, MD Travis Vandergriff, MD

21a. Atypical Vascular Lesion Jenna Beasley, MD Yasmine Humeda, MS3

21b. Organizing Thrombus Presenting as a Painful Skin Tumor Philip R. Cohen, MD Christof P. Erickson, MD Antoanella Calame, MD

22a. Angiolymphoid Hyperplasia with Eosinophilia Ardenne Martin Kelly Vogel, MD Jason L. Smith, MD

22b. Angiolymphoid Hyperplasia with Eosinophilia Terry Henges, PA Clay J. Cockerell, MD

23. Heparin-Induced Bullous Hemorrhagic Dermatosis Margaret E. Brown, MD Frank Jing, MS Huma A. Siddiqui, MD Jennifer Krejci- Manwaring, MD

24. Severe Esophageal Involvement in Cicatricial Pemphigoid Chelsea Harper, DO Jean Elizze Charles, DO Leah Persad, DO Stephen E. Weis, DO

25. Childhood Linear IgA Bullous Dermatosis Fred Ghali, MD Clay J. Cockerell, MD

26. Unusual Presentation of Pityriasis Lichenoides Chronica with Sarah DeVaro, BA Overlapping Features of Erythema Multiforme Stephen Squires, MD Dipti Anand, DO

10

27a. Pityriasis Lichenoides- Like Variant of Sarah DeVaro, BA Dipti Anand, MD Abna Soni, DO

27b. Patch Stage Mycosis Fungoides Giuseppe Cannata, MD Clay J. Cockerell, MD

28. Telangiectasia Eruptiva Macularis Perstans Variant Alison Black, MD of Mastocytosis Clay J. Cockerell, MD

29. Dermal Filler Foreign Body Reaction to Nivolumab Diana Losak, MD Immunotherapy Clay J. Cockerell, MD

30. Lambda Light Chain Multiple Myeloma and Primary Kavina Patel, BS Systemic Amyloidosis Zachary Gillooly, MD Margaret Brown, MD Robert T. Gilson, MD

31. Lichen Planopilaris with Rupture Simulating Folliculitis Giuseppe Cannata, MD Decalvans Clay J. Cockerell, MD

32. Acyclovir Resistant Anogenital Herpes Simplex in Jean Elizze Charles, an HIV patient with Pseudoepitheliomatous DO Hyperplasia Resembling Squamous Cell Carcinoma Stephen E. Weis, DO Gregory A. Hosler, MD, PhD

33. Spongiotic Viral Exanthem Fred Ghali, MD Clay J. Cockerell, MD

34. Disseminated Cryptococcosis Daniel Glade, BS Amreen Sitabkhan, MD Mary Shepard, MD

35. Childhood Endogenously Acquired Fred Ghali, MD Leishmaniasis Clay J. Cockerell, MD

11

DOUBLY RECURRENT NEVUS

Case No. 1 Care provider: Mark A. Cappel, MD Tampa, Florida Elleanor Swartz, PA-C Jacksonville, Florida

History: A 37-year-old male presented for a skin check and had a shave biopsy of a clinically atypical nevus on the right midback. The patient was seen 2.5 months later and the shave biopsy scar had re-pigmented centrally. An additional shave procedure was completed in an effort to remove this recurrent pigment. The patient was seen 1.5 months later, and the shave scar had again re-pigmented centrally. An elliptical excision with clear margins was then completed.

Physical Examination: At initial presentation on the right midback, there was an oval pigmented lesion composed of a central brown papule with a circumferential brown macular component (so-called ‘fried-egg nevus’). At the 2 subsequent visits, a darkly pigmented macule was present centrally within the shave scar.

Laboratory Data: Not applicable

Histopathology: 1st, shave biopsy: compound nevus with some features of a Clark’s nevus (mildly dysplastic nevus with junctional bridging and a lateral shoulder) and a congenital nevus (peri-adnexal dermal pattern). The nevus focally extended to the deep margin of the shave specimen. 2nd, shave removal: residual atypical compound nevus and scar, consistent with recurrent nevus. The nevus and scar extended to the deep margin of the shave specimen. 3rd, excision: residual atypical compound nevus and scar, consistent with recurrent nevus. The surgical margins were free of nevus.

Clinical Course: No recurrent pigment at the excision site.

Diagnosis: Recurrent / persistent nevus (twice).

Points of Emphasis: Melanocytic nevi can recur or persist if not completely surgically removed and are capable of mimicking malignant melanoma, both clinically and histologically. Nevi are more likely to recur if the original nevus was > 1cm in size, had positive margins, was located on the back, and was removed by a shave procedure. Nevi with a dermal component are the most common to recur, possibly due to residual melanocytes in the deep dermis around adnexal structures. Accurate determination of the original type of nevus from microscopic examination of the recurrent nevus is not always possible, particularly since the atypia in the recurrence appears to be independent of the type of

12 nevus that was originally biopsied. A helpful feature to clinically and histologically distinguish a recurrent nevus from melanoma is that generally the melanocytic proliferation is localized within the scar in recurrent nevi, whereas extension beyond the scar would raise more concern for melanoma.

References: 1. Arch Dermatol. 1975 Dec;111(12):1588-90. Pseudomelanoma: recurrent melanocytic nevus following partial surgical removal. Kornberg R, Ackerman AB. 2. J Am Acad Dermatol. 1987 Aug;17(2 Pt 1):285-92. Recurrent melanocytic nevi: clinical and histologic review of 175 cases. Park HK, Leonard DD, Arrington JH 3rd, Lund HZ. 3. Mod Pathol. 2009 May;22(5):611-7. Epub 2009 Mar 6 Recurrent nevus phenomenon: a clinicopathologic study of 357 cases and histologic comparison with melanoma with regression. King R1, Hayzen BA, Page RN, Googe PB, Zeagler D, Mihm MC Jr. 4. J Cutan Pathol. 2011 Jun;38(6):503-7. Epub 2011 Mar 1. Persistent melanocytic nevi: a review and analysis of 205 cases. Sommer LL1, Barcia SM, Clarke LE, Helm KF.

13

ATYPICAL MELANOCYTIC PROLIFERATION

Case No. 2 Care provider: Julia Accetta, MD Lauren Chen, BS Andrea Murina, MD New Orleans, Louisiana

History: 36 year old male with no significant past medical history presented for a full body skin exam. No personal history of skin cancer, history of prior biopsies consistent with dysplastic nevi. Family history of mother with melanoma.

Physical Examination: General: Well developed, well-nourished male Patient with Fitzpatrick Type I and multiple pink-brown nevi Face/scalp: Left parietal scalp 4 mm brown thin papule with some border irregularity and varying pigmentation. On dermoscopy, two greyish globules noted within lesion. Scattered monomorphous peach/brown thin nevoid papules on the scalp. Lymph: No cervical or submandibular lymphadenopathy

Laboratory Data: N/A

Histopathology: Sections show skin with a proliferation of atypical melanocytes at the basal layer of the epidermis and in the superficial dermis disposed as single cells and as small clusters, highlighted on Sox-10 and Melan-A stains, with prominent regression changes. There are melanocytes that demonstrate balloon cell change and marked pleomorphism. No obvious pagetoid upward migration or colonization of the adnexal structures is identified. No melanocytes are identified in angio-lymphatics on D2-40, CD31, or CD34 stains. All controls are adequate. Ki-67 labeling is unremarkable. Diagnosis: Atypical melanocytic proliferation. Note: The etiology is unknown. However, malignant melanoma of a metastatic origin is not completely ruled out. Margins examined are free of the tumor involvement.

Clinical Course: Because of the uncertain nature and lack of any other clinically identifiable primary lesions, it was recommended that the patient undergo screening. The patient was seen by an outside provider for wide excision of the lesion. Subsequent tissue analysis stated that there was focal residual atypical melanocytic proliferation with mild nuclear atypia and clear margins. No melanoma was identified. Patient has been followed with skin exams and has had no further concerning lesions to date.

Diagnosis: Atypical melanocytic proliferation

14

Points of Emphasis: This is a challenging case due to clinico-pathologic discordance. On clinical examination and dermoscopy, there appeared to be a 4 mm nevoid papule with atypical features suggestive of melanoma vs dysplastic nevus (atypical network, blue-gray blotches.) Histopathology revealed atypical melanocytic proliferation without well-formed nests and concern for metastatic melanoma. Ultimately no primary lesions were identified and the patient was treated with wide resection with clear margins and rule out of metastatic melanoma. Atypical intraepidermal melanocytic proliferation (AIMP) can be difficult to manage due these inconsistencies and the uncertainty of whether or not the lesion is malignant. In one study, 71 out of a total of 1127 AIMP biopsies were described pathologically as “atypical melanocytic proliferation” and 248/1127 as “cannot exclude melanoma/MIS”. Of those described as atypical melanocytic proliferation and cannot exclude melanoma/MIS, 92.96% and 92.74% were not melanoma/MIS after excision respectively (1). In another study, diagnostic change to melanoma occurred in 4.2% (13/306) AIMP lesions after review of the excisional histology. Risk factors that may lead to AIMP being upstaged to melanoma include location (head, neck, acral), lesions that extended to the base of initially biopsied specimens, using punch biopsy instead of shave biopsies, when melanoma was in the pathologist’s differential diagnosis for initially biopsied specimen, and finally when >40% of pigment remained after the preoperative biopsy (2). Some of the potential reasons behind why there is incongruity is described by Ensslin, et al. There is a lack of standardized diagnostic terminology, which makes it difficult to predict clinical behavior and guide treatment (3). There are multiple terms that represent histologically ambiguous melanocytic proliferation and these terms vary by institution. At the XXIX Symposium of International Society of Dermatopathology in 2008, international experts were asked to evaluate 57 cases previously deemed MELTUMP in an effort to obtain histopathologic criteria that could classify atypical melanocytic tumors as benign or malignant. Ultimately there was no consensus and agreement in only 30% of cases. In addition, pathologists incorrectly classified 53% of cases which had a favorable outcome as malignant, and 27% of cases which had an unfavorable outcome (local recurrence, metastasis, death) as benign (4). Although AIMP and other nomenclature may be unavoidable, further steps can be taken to minimize ambiguity. According to Blank et al., complete excisional biopsies of suspicious pigmented lesions and clinical information sharing with pathologist (lesion size, appearance, history, full vs partial biopsy) enhance clinicopathologic correlation. Dermoscopy and confocal microscopy also help in diagnosis. Ancillary pathology techniques such as IHC, cytogenetic studies and gene expression assays have shown potential in improving diagnostic precision. Overall, effective communication between dermatologists and pathologists can help minimize uncertainty and improve management of atypical melanocytic lesions.

References: (At least two recent pertinent published references) 1. Blank, N. R., Hibler, B. P., Tattersall, I. W., Ensslin, C. J., Lee, E. H., Dusza, S. W., … Rossi, A. M. (2019). Melanoma and melanoma in-situ diagnosis after excision of atypical intraepidermal melanocytic proliferation: A retrospective cross-sectional

15

analysis. Journal of the American Academy of Dermatology, 80(5), 1403–1409. doi: 10.1016/j.jaad.2019.01.005 2. Zhang, J., Miller, C. J., Sobanko, J. F., Shin, T. M., & Etzkorn, J. R. (2016). Diagnostic Change From Atypical Intraepidermal Melanocytic Proliferation to Melanoma After Conventional Excision—A Single Academic Institution Cross- Sectional Study. Dermatologic , 42(10), 1147–1154. doi: 10.1097/dss.0000000000000877 3. Ensslin, C. J., Hibler, B. P., Lee, E. H., Nehal, K. S., Busam, K. J., & Rossi, A. M. (2018). Atypical Melanocytic Proliferations. Dermatologic Surgery, 44(2), 159–174. doi: 10.1097/dss.0000000000001367 4. Cerroni, L., Barnhill, R., Elder, D., Gottlieb, G., Heenan, P., Kutzner, H., … Kerl, H. (2010). Melanocytic Tumors of Uncertain Malignant Potential. The American Journal of Surgical Pathology, 34(3), 314–326. doi: 10.1097/pas.0b013e3181cf7fa0 5. Jew, O., Miller, C. J., Shin, T. M., Sobanko, J. F., & Etzkorn, J. R. (2018). Diagnostic Change From Atypical Intraepidermal Melanocytic Proliferation to Melanoma is More Likely When Clinically Visible Residual Pigment Remains After Biopsy. Dermatologic Surgery, 44(7), 1024–1026. doi: 10.1097/dss.0000000000001387 6. Hazan, C., Dusza, S. W., Delgado, R., Busam, K. J., Halpern, A. C., & Nehal, K. S. (2008). Staged excision for lentigo maligna and lentigo maligna melanoma: A retrospective analysis of 117 cases. Journal of the American Academy of Dermatology, 58(1), 142–148. doi: 10.1016/j.jaad.2007.09.023 7. Möller, M. G., Pappas-Politis, E., Zager, J. S., Santiago, L. A., Yu, D., Prakash, A., … Cruse, C. W. (2008). Surgical Management of Melanoma-In-Situ Using a Staged Marginal and Central Excision Technique. Annals of Surgical , 16(6), 1526–1536. doi: 10.1245/s10434-008-0239-x

16

Case No: 3

Melanoma In Situ Histologically Simulating “Dysplastic” Nevus

Care provider: Terry Henges, PA-C Clay J. Cockerell, MD Chapel Hill, North Carolina Dallas, Texas

History: A 54 year old Caucasian man presented with a pigmented lesion of the left lower back that had been present for years and slowly growing.

Physical Examination: A large, asymmetrical irregular pigmented lesion was noted with some whitish areas suggestive of regression

Laboratory Data: N/A

Histopathology: There was a proliferation of melanocytes arranged as nests but also as solitary units with some atypia but not strikingly so. Relatively minimal pagetoid spread was noted and there were no mitoses. The biopsy was taken in parte so assessment of breadth, symmetry and circumscription could not be performed. The original diagnosis was that of a compound “dysplastic” nevus. Because of clinical concern about the diagnosis, a clinical photograph was submitted to evaluate in the context of the histology. Additional sections were cut and evaluated leading to a revised diagnosis of melanoma in situ arising in association with a dysplastic nevus.

Diagnosis: Melanoma in situ arising in a “dysplastic” nevus

Clinical Course: The patient underwent surgical excision.

Points of Emphasis: While most melanomas are readily diagnosed histologically, there can be significant overlap between some benign pigmented lesion causing diagnostic difficulty. That difficulty can be compounded when either a non-representative lesion is sampled, especially when the lesion is sampled by punch or narrow, superficial shave technique. When lesions are particularly broad, a broad saucerization or incisional biopsy is strongly recommended. As many melanomas arise in association with nevi, a partial biopsy may sample the benign component and fail to sample the malignant one. In cases where there is discordance between the clinical and histologic diagnosis, reviewing clinical photographs can be valuable as in this case which led to additional deeper sections being evaluated and a revised diagnosis being rendered. It behooves clinicians to remain vigilant and where the clinical diagnosis does not agree with the histologic one, a communication to the dermatopathologist is in order to avoid misdiagnosis.

17

References: Glazer A and Cockerell CJ. Histopathologic discordance in melanoma can have substantial impacts on patient care. SKIN 3:2, 2019.

18

Case No: 4 Desmoplastic Melanoma

Care provider: Jason L. Smith, MD Rome, Georgia

History: 81 year-old male seen at his usual 6 month follow-up for actinic keratosis and extensive solar radiation. Upon exam, a smooth tan, pink papule surrounded by irregular tan brown macule was noted on his right mid cheek. Patient was not aware of how long this lesion had been present.

Physical Examination: 2.7 cm irregular brown macule on right mid cheek with a 5 mm smooth tan pink papule in the center.

Laboratory Data: N/A

Histopathology: Sections demonstrate an atypical junctional melanocytic proliferation comprised predominantly of confluent atypical single cells with marked cytologic atypia, focal retraction at the dermal epidermal junction, and confluent extension down hair follicles. Pagetoid spread is readily apparent on routine stains and confirmed on Melan- A immunohistochemical stain with appropriate control. The dermis appears scarlike from low-power with a few small lymphoid aggregates and focal collections of dense melanin pigment. Upon closer examination, both epithelioid and spindle cell melanocytes with mild to moderate cytologic atypia, hyperchromatic nuclei, and intracytoplasmic melanin pigment is present in a few of the cells. Scattered lymphocytes are also present intermixed amongst the discohesive single cell melanocytes. A few small melanocytic nests are present in the superficial dermis. Immunohistochemical stains were performed to further classify these cells. Melan A and HMB-45 highlight the intraepithelial melanocytes comprising the junctional component, however the dermal cells are completely negative. SOX-10 highlight both the junctional and single cell dermal melanocytes supporting that these are indeed melanocytic in origin. KI-67 proliferation rate is relatively low showing a 5-10 percent proliferation index. This combination of histopathological and immunohistochemical findings are best classified as desmoplastic melanoma with a depth of at least 1.1 mm. Mitotic figures are absent. Lymphovascular invasion, regression, and satellitosis are also absent. Melanoma in situ extends to both peripheral margins and the invasive desmoplastic melanoma involves the entire deep margin.

Immunohistochemical stains: Melan A-highlight the intraepithelial melanocytes comprising the junctional component, however the dermal cells are completely negative. HMB-45- highlight the intraepithelial melanocytes comprising the junctional component, however the dermal cells are completely negative. SOX-10-highlight both the junctional and single cell dermal melanocytes supporting that these are indeed melanocytic in origin. KI-67-proliferation rate is relatively low showing a 5-10 percent proliferation index

19

Clinical Course: Complete resection by head/neck surgeon

Diagnosis: Desmoplastic Melanoma Thickness= at least 1.1 mm Ulceration: not present Clark’s Level: at least Clark’s Level III

Points of Emphasis: Suspicious raised papule with very little pigment in a lesion of lentigo maligna

References: 1. Chen, Lucy L. et al. Desmoplastic Melanoma: A Review. J Am Acad Dermatology. 2013 May: 68 (5): 825-833. 2. Hollmig, Tyler S. et al. Spindle Cell Neoplasms Encountered in Dermatologic Surgery: A Review. Dermatologic Surgery: June 2012 Volume 38 Issue 6- p. 825-850.

20

Case No: 5

MELANOMA MASQUERADING AS A DERMATOFIBROMA

Care provider: Faryal Siddiqui, MD Francis Sharkey, MD Whitney High, MD Sandra Osswald, MD Daniel Glade, BS San Antonio, Texas

History: 60-year-old male with past medical history of liver transplant in July 2015, immunosuppressed with tacrolimus, presents to the dermatology clinic for a complete skin exam. Patient reports a 3-year history of a pink bump on his right medial ankle. The lesion bleeds spontaneously and is tender. Patient reports recent itchiness to the shin and calf adjacent to the lesion on the right leg. Patient denies unintentional weight loss, night sweats, fevers or chills. Patient has no personal or family history of skin cancer.

Physical Examination: Exam revealed a well-nourished, well-developed male with skin type 4 and a 4-mm pink dome shaped firm papule on the right medial malleolus.

Laboratory Data: None

Histopathology: A shave biopsy was performed. Hematoxylin and eosin stains demonstrated malignant proliferation of atypical melanocytes with marked mitotic activity. At least 11 mitotic figures were identified in the dermal component. Additional stains revealed S100/Sox10 positive cells with Verocay body-like formations. Mart-1 showed rare positive scattered cells. KI-67 demonstrated a slightly elevated proliferation index in some of the atypical islands. P16 expression was absent. Atypical cells were broadly present at the deep surgical edges.

Clinical Course: Patient underwent wide local excision with skin graft to cover the medial malleolar excision site. Right groin sentinel lymph node biopsy revealed no malignancy. Patient continues to receive follow-up in our clinic.

Diagnosis: Melanoma

Points of Emphasis: As in the case of our patient, differentiating between melanomas and benign neoplasms such as dermatofibromas can be difficult as they may appear similar clinically. Theoretically both could occur concomitantly in a collision tumors, though none have been reported. History of spontaneous bleeding, itchiness, and absence of prior trauma make dermatofibroma less likely and merits a biopsy. Other neoplasms that can resemble dermatofibromas include Spitz nevus, desmoplastic melanoma, and amelanotic melanoma. Dermatofibromas tend to show positive S-100 staining focally, whereas melanoma usually have diffuse, strong cytoplasmic and nuclear staining. Most commonly seen in schwannomas, Verocay body pattern refers to

21 the rippled pattern produced by alternating strands of stroma and epithelial cell cords. This finding is non-specific as it may also appear in fibrohistiocystic, mesenchymal, adnexal, epithelial, and melanocytic neoplasms.

References: 1. Biswas, A., Setia, N. and Bhawan, J. (2011). Cutaneous Neoplasms with Prominent Verocay Body–Like Structures: The So-called “Rippled Pattern”. The American Journal of Dermatopathology, 33(6), pp.539-550. 2. Kovach, B. and Boyd, A. (2007). Melanoma associated with a dermatofibroma. Journal of Cutaneous Pathology, 34(5), pp.420-422.

22

Case No: 6a

LINEAR UNILATERAL BASALOID FOLLICULAR PROLIFERATION

Care provider: Mark A. Cappel, MD Kyle Bartlett, MD Tampa, FL

History: A 32-year-old male presented with linear unilateral macular lesions for years on the right upper extremity. Within the linear macular lesions, occasional papules began developing. The papular lesions were biopsied. The patient is otherwise healthy and has no developmental abnormalities.

Physical Examination: Linear erythematous plaque on the dorsal right index finger Linear punctate pits on the right mid proximal palm Waxy papule on the right ventral wrist Linear hypopigmented macules involving the right dorsal forearm, and right lateral elbow, arm, and shoulder

Laboratory Data: Not applicable

Histopathology: 5 shave biopsies show similar features, with multi-nodular and racemose collections of atypical basaloid follicular cells. Some of the basaloid proliferations appear to be arising from hair follicles and the epidermis. There are also some basaloid nodules within the dermis, with no apparent connection to hair follicles or the epidermis. Peripheral palisading and myxoid change is focally seen. The surrounding tumor stroma appears scant. BerEP4 is positive throughout the basaloid proliferations. CD10 and BCL2 focally stain the areas that most resemble basal cell carcinoma on H&E. CK20 stains occasional Merkel cells in the areas that most resemble basaloid follicular hamartoma on H&E.

Clinical Course: Treating lesions that clinically appear and behave as basal cell carcinoma.

Diagnosis: Linear unilateral basaloid follicular proliferation, consistent with multiple segmental infundibulocystic basal cell carcinomas.

Points of Emphasis: Pathologically, this case highlights the challenge of distinguishing basaloid follicular proliferations with overlapping histologic features. Trichoepithelioma is most characterized by an abundant highly fibrocytic stroma and germinative follicular papillae and bulbs. Basaloid follicular hamartoma is most characterized by basaloid cords and strands centered on malformed and distorted hair follicles. Infundibulocystic basal cell carcinoma is most characterized by an atypical basaloid proliferation involving both hair follicles and the interfollicular dermis.

23

Clinically, this case demonstrates a linear unilateral pattern of distribution of multiple basaloid follicular proliferations. The term “linear basal cell carcinoma” refers to a basal cell with a length-to-width ratio of at least 3:1, however the linearity in this case suggests a segmental or mosaic genetic aberration. There have been prior reports in the literature of unilateral: infundibulocystic basal cell carcinomas, basal cell nevus, nevoid basal cell carcinoma syndrome, and basaloid follicular hamartoma. This case is presented for discussion of this variable diagnostic terminology and suggested treatment approach.

References: 1. Arch Dermatol. 1999 Oct;135(10):1227-35. Multiple hereditary infundibulocystic basal cell carcinomas: a genodermatosis different from nevoid basal cell carcinoma syndrome. Requena L1, Fariña MC, Robledo M, Sangueza OP, Sanchez E, Villanueva A, Marquina A, Tamarit R. 2. Dermatol Surg. 2003 Apr;29(4):436-9. Multiple infundibulocystic basal cell carcinomas: case report of unique unilateral presentation. Herman AR1, Busam KJ, Greenberg RA, Nehal KS. 3. J Cutan Pathol. 2007 Jan;34(1):65-70. Congenital linear unilateral basal cell nevus: a case report with patched gene molecular studies. Brailey LL1, Davis T, Kolker SE, Murry TC, Thomas D, Bale AE, Ruhoy SM. 4. Cutis. 2006 Aug;78(2):122-4. Linear unilateral basal cell nevus: a benign follicular hamartoma simulating multiple basal cell carcinomas. Shumaker PR1, Lane K, Harford R. 5. J Am Acad Dermatol. 1986 Nov;15(5 Pt 1):1023-30. Nevoid basal cell carcinoma syndrome. A review and case report of a patient with unilateral basal cell nevus syndrome. 6. Pediatr Dermatol. 2017 Jul;34(4):e196-e200. doi: 10.1111/pde.13185. Epub 2017 Jun 29. Linear Unilateral Basaloid Follicular Hamartoma Following Blaschko's Lines: Two Case Reports and Review of the Literature. López Jiménez EC1, Grau C2, Islas Norris D1, Montenegro Dámaso T3, Azón A2, Borrego Hernando L1. 7. J Dermatol. 2017 Nov;44(11):e278-e279. doi: 10.1111/1346-8138.13959. Epub 2017 Jul 10. Unilateral linear basaloid follicular hamartoma: Its diagnosis and successful treatment with surgical excision and adjuvant dermabrasion. 8. J Cutan Med Surg. 2005 Dec;9(6):336-40. Unilateral Basal cell carcinomas: an unusual entity treated with photodynamic . Lane JE1, Allen JH, Lane TN, Lesher JL Jr.

24

Case No: 6b

Basal Cell Nevus Syndrome in a 9 month Infant

Care provider: Jeffrey Scott Harbold Jr, DO McAllen, Texas

History: We present a 9 month Hispanic male with a progressive asymptomatic linear atrophic plaque on the flexor surface of the left wrist extending to the left upper arm. The lesion was present at birth and has progressed in a linear pattern towards the proximal arm. The patient has a history of pyloric stenosis s/p pylorectomy, macrocephaly, and patent foramen ovale/peripheral pulmonic stenosis. There was no significant dermatologic family history reported.

Physical Examination: This patient presented with a linear atrophic plaque on the flexor surface of the left wrist extending to the left upper arm. In addition, the patient was noted to have a cleft lip with intact palate, craniomegaly with frontal bossing, an extranumerary digit of the right foot, and plantar pits.

Laboratory Data: PTCH1 testing positive. The patient is heterozygous for the pathologic variant, c.2515C>T (p.Gln839*) of the PTCH1 gene.

Histopathology: Sections demonstrate basaloid neoplasm with stromal retraction with peripheral palisading. There is no evidence of solar elastosis in the reticular dermis.

Clinical Course: Due to the size of the lesion and the age of the patient, topical management was recommended over surgical excision and patient was started topical imiquimod. The child was referred to pediatric where an MRI head was ordered to evaluate for meduloblastoma. Patient was also referred to ophthalmology and pediatric for further evaluation. The patient was referred to , where he underwent surgical treatment of the polydactyly. Strong emphasis was placed on photoprotection and routine skin check ups.

Diagnosis: Basal Cell Nevus Syndrome, Gorlin Syndrome

Points of Emphasis: This case demonstrates an early onset and a somewhat usually morphology of Basal Cell Nevus Syndrome. The early diagnosis in this case will allow for close surveillance of this young patient.

25

References: 1. Scott C. Bresler, Bonnie L. Padwa, and Scott R. Granter. Nevoid Basal Cell Carcinoma Syndrome (Gorlin Syndrome) Head Neck Pathology. 2016 Jun; 10(2): 119–124 [PMID: 26971503] 2. Foulkes WD, Kamihara J, Evans DGR, et al. Cancer Surveillance in Gorlin Syndrome and Rhabdoid Tumor Predisposition Syndrome. Clin Cancer Res 2017; 23:e62. [PMID:28620006]

26

Case No: 6c

Basal Cell Carcinoma Transformation in a Nevus Sebaceus

Care provider: Lucy Chao, MD Abida Kadi, MD Leah Jacob, MD New Orleans, LA

History: The patient is a 43 year-old Hispanic woman who presents with an enlarging lesion on her scalp. She states that the lesion has been present since birth but started growing in size two months ago. The lesion becomes irritated when she combs her hair or scratches her scalp, which results in mild pain and bleeding. She also endorses mild pruritus. Denies using any topical medications on the lesion.

Physical Examination: General: well developed, well nourished female in no acute distress. Skin: frontal scalp with yellowish-pink verrucous stuck-on plaque, medial side is pedunculated with heme crusting, lateral side with two superimposed hyperpigmented papules.

Laboratory Data: N/A

Histopathology: 1) Skin, scalp, biopsy: pigmented basal cell carcinoma, deep margin involved. Comment: There appears to be a background of nevus sebaceous with associated hamartomatous lesions. 2) Skin, scalp, biopsy: nevus sebaceus with atypical sclerosing epithelial neoplasm and atypical basal cell hyperplasia (possible incipient basal cell carcinoma), margin involved.

Clinical Course: During the patient’s initial evaluation, two shave biopsies were performed. One biopsy was consistent with pigmented basal cell carcinoma arising within a nevus sebaceous. The other biopsy showed a nevus sebaceous with atypical sclerosing epithelial neoplasm and atypical basal cell hyperplasia. Given the size of the lesion and location on the scalp, the patient was referred to plastic surgery for complete excision.

Diagnosis: Basal cell carcinoma transformation in a nevus sebaceus

Points of Emphasis: Secondary tumors, both benign and malignant, commonly arise within a nevus sebaceous. The most common malignant tumor to develop is basal cell carcinoma, which arises at a rate of approximately 1%. Signs of secondary tumor development include pigmented, pedunculated, eroded, filiform and hyperkeratotic changes.

27

References: 1. Hsu MC, Liau JY, Hong JL, et al. Secondary neoplasms arising from nevus sebaceous: A retrospective study of 450 cases in Taiwan. J Dermatol. 2016 Feb;43(2):175-80. 2. Moody MN, Landau JM, Goldberg LH. Nevus sebaceous revisited. Pediatr Dermatol. 2012 Jan-Feb;29(1):15-23.

28

Case No: 7a ATYPICAL FIBROXANTHOMA

Care provider: Jason L. Smith, MD Rome, Georgia

History: 72 year old male presents with an enlarging, bleeding, and painful lesion on left ear that started 2 months ago.

Physical Examination: 1.5 cm pink erosive tumor nodule on left mid helical rim; no palpable lymphadenopathy on head and neck

Laboratory Data: N/A

Histopathology: nodular epithelioid and spindle cell neoplasm with marked pleomorphism, bizarre multinucleated giant cells and atypical mitotic figures. The epidermis is markedly ulcerated in the center of the lesion while forming a collarette at the periphery of the nodular proliferation. The dermis is entirely filled with the atypical proliferation with cells demonstrating marked pleomorphism described with copious amounts of eosinophilic cytoplasm.

Immunohistochemical stains: CD10- strongly positive CD31-negative CK-5/6 negative p40- negative S100-negative desmin-negative SOX-10-negative

Clinical Course: Complete excision with no residual tumor

Diagnosis: Atypical Fibroxanthoma

Points of Emphasis: Rapidly growing tumor nodule on the ear consistent with AFX while pleomorphic dermal sarcoma was considered

References: 1. Zwald MD, Fiona, Maher MD, Ian A., 2019, Nov, Atypical Fibroxanthoma, www.uptodate.com 2. Elwood MD, Hillary R, 2019, April 18, Skin Non-Melanocytic Tumor: Fibrous, fibrohistiocytic and myofibroblastic neoplasms

29

Case No: 7b

PLEOMORPHIC DERMAL SARCOMA

Care provider: Hope Retif MS-II Taylor Smith MD Keith Harris MD Rome, Georgia

History: 84-year-old white/Caucasian male with a history of squamous cell carcinoma and basal cell carcinoma presents with a raised red lesion on his scalp. The lesion was present for 2-3 months and occasionally bled. Denies pruritus. No history of HIV or immunosuppression.

Significant Past Medical History: actinic keratosis, basal cell carcinoma and squamous cell carcinoma of neck, squamous cell carcinoma of trunk, squamous cell carcinoma in situ of left cheek, extensive exposure to solar radiation, congestive heart failure, and a left bundle branch block.

Physical Exam: pertinent findings - right anterior vertex scalp 1.1 cm red dome shaped nodule

Histopathology: Shave biopsy of right anterior vertex scalp – fascicles of atypical spindled and multinucleated cells with pleomorphic and hyperchromatic nuclei are present throughout the dermis. Atypical cells are highlighted with a CD10 stain. These cells do not stain for S100, Melan-A, CK AE1-AE3, or CK5/6.

Clinical Course: The patient was referred to Mohs surgery on 4/2018 for complete excision with clear margins of the atypical fibroxanthoma. On 9/23/2019 the patient complained of a new asymptomatic lesion on his right vertex scalp that had been present 8-10 weeks. A 1.5cm light red subcutaneous nodule was noted on exam roughly 2 cm posterior to the original atypical fibroxanthoma. A 4mm punch biopsy of the lesion showed fascicles of atypical spindled cells with hyperchromatic nuclei present throughout the dermis and extending into the subcutis to the base of the specimen. The cells showed patchy positivity for CD10 and are negative for CD34, p63, CK5/6, CK AE1-3, SOX10, Melan-A, and S100. These results argue against dermatofibrosarcoma protuberans. S100 also highlights a nerve twig surrounded by tumor. Malignant spindle cell tumor compatible with pleomorphic dermal sarcoma extending to edge. The lesion was completely excised and grafted.

Diagnosis: Atypical Fibroxanthoma (AFX) and Pleomorphic Dermal Sarcoma (PDS)

Points of Emphasis: While AFX and PDS can have morphological similarities such as spindle cell histology, atypical mitotic figures and CD10 positivity, PDS is defined by

30 subcutis involvement, presence of lymphovascular, perineural invasion or necrosis. Surgical excision is the first line of management. Mohs surgery should have cleared the patient of his first AFX. Over one year later the patient develops a new PDS very close to the original site. Was the surgical margin somehow missed? Is this a local occurrence of a different tumor given slight variation in pleomorphism of cells, or is this a recurrence of the original lesion?

References: Tolkachjov, Stanislav N et al. (2018). Atypical fibroxanthoma: systematic review and meta-analysis of treatment with Mohs micrographic surgery or excision. Journal of the American Academy of Dermatology, Volume 79, Issue5, 929-934.e6 Griewank, K. G., Wiesner, T., & Murali, R. (2017). Atypical fibroxanthoma and pleomorphic dermal sarcoma harbor frequent NOTCH1/2 and FAT1 mutations and similar DNA copy number alteration profiles. Modern Pathology, 31(3), 418–428. doi: 10.1038/modpathol.2017.146 Klebanov, N., Hoang, M. P., & Reddy, B. Y. (2018). Pleomorphic Dermal Sarcoma of the Scalp. Cureus. doi: 10.7759/cureus.2979 Soleymani, T., Aasi, S. Z., Novoa, R., & Hollmig, S. T. (2019). Atypical Fibroxanthoma and Pleomorphic Dermal Sarcoma. Dermatologic Clinics, 37(3), 253–259. doi: 10.1016/j.det.2019.02.001

31

Case No: 7c

Pleomorphic Dermal Sarcoma

Care provider: John Adnot, MD Wade Smith, MD Clay J. Cockerell, MD Dallas, Texas

History: A 79 year old Caucasian woman presented with a growth of her right leg that had been present for over 6 months. The lesion had been increasing in size over the previous 3 months. She had a past history of lupus erythematosus, hypertension and osteoarthritis.

Physical Examination: A 6cm exophytic superficially eroded tumor was present on the right lower leg. There was erythema and induration extending for a number of centimeters beyond the neoplasm.

Laboratory Data: N/A

Histopathology: There was a diffuse spindle cell neoplasm comprised of atypical spindle cells arranged in fascicles. There was superficial erosion. Numerous atypical mitoses were present.

Immunohistochemical stains were positive for CD-10 and for Procollagen 1. Stains for CK and Sox-10 were negative. The diagnosis of atypical fibroxanthoma was rendered.

Upon discussion with the Mohs surgeon scheduled to perform the re-excision, it was noted that this was a very large neoplasm and only a small portion was taken as a biopsy. When reviewing the biopsy specimen with clinical photographs, a revised diagnosis of pleomorphic dermal sarcoma was made.

Two additional punch biopsies were performed of the indurated area surrounding the neoplasm to ensure that this did not represent local spread both of which revealed only stasis change and inflammation.

Diagnosis: Pleomorphic dermal sarcoma

Clinical Course: The patient underwent wide local excision which resulted in clear margins. She is undergoing evaluation to exclude metastasis.

Points of Emphasis: Pleomorphic dermal sarcoma (PDS), formerly known as malignant fibrous histiocytoma, is a rare mesenchymal tissue tumor. Distinguishing PDS from atypical fibroxanthoma (AFX) is difficult as the 2 are clinically and histologically similar. It is debated whether PDS represents a separate clinical entity from AFX or it is

32 merely a more aggressive deeper variant. As diffuse CD10 positivity is observed in both AFX and PDS, CD10 positivity alone cannot be used to make the diagnosis of PDS. The most valuable feature is involvement of the deep subcutis and large size of the tumor. As such, and as in this case, a superficial biopsy will not allow for the distinction between the 2 entities and clinical correlation is required. The treatment for AFX is wide local excision and Mohs micrographic surgery. Less is known overall regarding PDS management given the tumor’s rarity although complete surgical excision is the first line option. In spite of aggressive surgery, there is a 20% local recurrence rate. Lesions of the soft tissue that were previously known as MFH have traditionally been treated by with wide excision and in some cases, amputation. References: Klebanov N, Hoang M, Reddy B. Pleomorphic Dermal Sarcoma of the Scalp Cureus. 2018 Jul; 10(7): e2979.

33

Case No: 8a SARCOMATOID SQUAMOUS CELL CARCINOMA

Care provider: Jason L. Smith, MD Rome, Georgia

History: 82 year-old male presents with a star like lesion on the right frontal scalp for several months. Patient has had extensive solar radiation and history of multiple skin cancer removals on the head and neck area.

Significant Past Medical History: for SCC on frontal scalp within the past year.

Physical Examination: 6 mm hemorrhagic and translucent scar like papule on right frontal scalp.

Laboratory Data: N/A

Histopathology: Biopsy from 03/26/2019: Sections of the shave show neoplastic basilar keratinocyte atypia, hyperkeratosis, and extensive dermal fibrosis, both well- established as well as with cellular, organizing areas. Some of the spindle cells within the organizing area fibrosis are plump and mildly atypical. Immunohistochemical stains for S100 and SOX-10 were performed to rule out the possibility of an atypical spindle cell melanocytic proliferation. SOX-10 is negative, and S100 is positive within dermal dendritic cells and not within the mildly atypical spindle cells. The findings are consistent with an inflamed scar with both well organized and organizing areas. There is no evidence of malignancy. Biopsy from 06/17/2019: Sections demonstrate a focally ulcerated and markedly inflamed shave of skin. Nonulcerated epidermis demonstrates mild to moderate acanthosis. Within the dermis is a diffuse infiltrate of plasma cells and lymphocytes, as well as a mildly to moderately cellular spindle cell proliferation. Most of the spindle cells have mildly enlarged nuclei, with occasional cells demonstrating moderate nuclear enlargement and prominent nucleoli. Mitotic activity is very difficult to discern due to the heavy inflammatory infiltrate. Interface changes are focally noted along the epidermis, with a few scattered necrotic keratinocytes present along the basal cell layer. Immunohistochemical stains were performed to better characterize the spindle cell proliferation, and results are reported below. A few of the atypical spindle cells stain positively for pancytokeratin and p63. The stains also highlight a few solid nests of atypical cells within the dermis.

Immunohistochemical stains: Biopsy from 03/26/2019: S100- positive within dermal dendritic cells and not within the mildly atypical spindle cells SOX-10- negative Biopsy from 06/17/2019: Negative stains: CK 5/6, S100, CD34, S100, HHV-8 Positive stains: CD68, CD10 Focally positive stains: Pancytokeratin, p63 Equivocal results: CD31

34

Clinical Course: Previous biopsy sites healed at one month but recurred at 3 months. Re-biopsy of hemorrhagic crusted papule. Patient consulted with radiation oncology and proceeded with radiation therapy.

Diagnosis: Sarcomatoid Squamous Cell Carcinoma

Points of Emphasis: Sarcomatoid Squamous Cell Carcinoma, mimicking a scar

References: 1. Hollmig, Tyler. S. MD. Et al. 2012 June. Dermatologic Surgery: Spindle Cell Neoplasms Encountered in Dermatologic Surgery: A Review. Volume 38-Issue 6- p 825-850. 2. Kostopoulos, Tassia C. MD. Et al. 2019 August. Dermatologic Surgery: Cutaneous Sarcomatoid Squamous Cell Carcinoma of the Ear and the Use of Immunohistochemical Staining as a Diagnostic Aid. Volume 45- Issue 8- p 1100- 1102.

35

Case No: 8b CUTANEOUS SQUAMO-MELANOCYTIC TUMOR

Care provider: Jason L. Smith, MD Keith R. Harris, MD Rome, Georgia

History: 80 year-old male presents with multiple lesions on his scalp with a history of extensive solar radiation. Patient is concerned with lesion on right mid scalp that has been enlarging for several months. Patient has had multiple BCC’s & SCC’s on head and neck requiring MOH’s surgery.

Physical Examination: 4 x 2.8 cm red tumor plaque and nodules on right mid scalp

Laboratory Data: N/A

Histopathology: Consultation by Dr. Christopher D.M. Fletcher, M.D. at Harvard in Boston, an expert in cutatious soft tissue writes as follows: “Discernibly arising from overlying dysplastic squamous epithelium, there is a component of this tumor showing features of an invasive poorly-differentiated squamous cell carcinoma with just focal keratinization. This component is strikingly positive for pan-keratin, CK5, and p63. However, in addition, there is a multifocally admixed population of spindle cells having mildly hyperchromatic tapering nuclei and indistinct pale cytoplasm, arranged in fascicles and showing scattered mitotic figures. There is no evident overlying junctional component, but the latter is difficult to assess in the setting of diffuse ulceration. This spindle cell component is multifocally positive for S100 protein and SOX10, while MART-1 is negative. The immunostains show that the squamous and melanocytic components are intimately admixed almost throughout the mass. Lesions such as this are known as cutaneous squamo-melanocytic tumor. Despite the worrisome morphologic components, the majority of these lesions seem not to be associated with aggressive behavior, which is somewhat unexpected, and it is probably best to be cautious regarding the prognosis, since there are less than 30 reported such cases. Ultrastructural data suggests that these are truly biphenotypic neoplasms (in which the cells show both lines of differentiation), rather than this being some unusual kind of collision tumor.”

Immunohistochemical stains: Pan-keratin-strikingly positive CK5- strikingly positive p63- strikingly positive S100-multifocally positive SOX10-multifocally positive MART-1- negative

Depth/stage: at least 3.2 mm, ulcerated (at least pT3b) Mitotic rate: 6/mm2

36

Angiolymphatic invasion: Not Idenitified Perineural Invasion: Not Identified Deep margin: Involved Peripheral margin: Involved

Clinical Course: Surgical resection by surgical oncologist with full thickness skin graft by plastic surgery

Diagnosis: Cutaneous Squamo-Melanocytic Tumor

Points of Emphasis: Rare tumor on mid scalp with two morphologically distinct neoplasms in close proximity. Discussion points might include theories of development such as a collision tumor vs. a tumor of dual/divergent differentiation.

References: 1. Wang HW et al. An uncommon malignant cutaneous squamomelanocytic tumor. Exp. Ther Med. 2013 Mar: 5(3): 897-901. 2. Kochoumian, E et al. Tumor with the features of both squamous cell carcinoma and melanoma (melanocarcinoma). Indian Dermatology Online Journal 2015:6(3):217-219.

37

Case No: 9a

LAMOTRIGINE-INDUCED CUTANEOUS CD 30+ ANAPLASTIC LARGE CELL LYMPHOMA

Care provider: Dipti Anand MD Aditya Sood M1 Caitlin Clealand, P.A.-C Abha Soni, DO Atlanta, Georgia

History: 68-year-old Caucasian female presents with a generalized pruritic rash plaque involving her trunk, upper extremities, and thighs for six months. The patient’s past medical history is significant for epilepsy and use of lamotrigine. She reports worsening of the rash with dose increments of lamotrigine, but its persistence despite discontinuation of the medication, and a course of steroids.

Physical Examination: Erythematous, slightly ill-defined, smooth pink plaques involving the chest, abdomen, back, arms and thighs. Clinical differential diagnoses included eczema, urticaria, psoriasis, contact dermatitis, and tinea.

Histopathology: Punch biopsy from the left abdomen showed a superficial & deep dermal perivascular & interstitial atypical dermal lymphoid infiltrate, without epidermotropism. The lymphoid cells were pleomorphic, enlarged, & hyperchromatic with irregular nuclear contours, and occasional conspicuous nucleoli. Scattered mitoses and crush artifact were seen. Atypical interstitial lymphoid cells were CD2+ CD4+ CD5+ CD30+, with CD3, CD8 and CD20 highlighting normal appearing perivascular T and B- lymphocytes. Additionally, EBER in situ hybridization was negative for Epstein-Barr mRNA. CD1a, TIA-1, CD34, CD117 and myeloperoxidase were also negative. A gene rearrangement study from the left mid abdomen was positive for T cell receptor gamma gene rearrangement and negative for B-cell gene rearrangement.

Diagnosis: Cutaneous CD30+ T-cell lymphoproliferative neoplasm, consistent with CD30+ Anaplastic large cell lymphoma (ALCL)

Clinical Course: Patient’s systemic staging for lymphoma was negative. Complete blood count (CBC) showed mild anemia, with no peripheral blood eosinophilia or .

Points of Emphasis: T-cell lymphomatoid drug reactions are described as T-cell cutaneous lymphocytic infiltrates induced by pharmacological therapy. These reactions histologically mimic cutaneous T-cell lymphoproliferative neoplasms, and may demonstrate CD30+ cells. Absence of clonality and clinical improvement or complete regression of the eruption upon drug cessation or modulation, can help to distinguish pseudolymphomatoid drug reactions from cutaneous T-cell lymphomas. A wide variety and class of drugs have been implicated in induction of lymphomatoid drug reactions. These include anticonvulsants like Phenytoin, Phenobarbital, Carbamazepine;

38 antidepressants like Sertraline, Fluoxetine; antihistamines like Hydroxyzine; antihypertensives like Lisinopril, Metoprolol, Amlodipine, Furosemide; immunomodulators like Prednisone, Tacrolimus; and chemotherapeutics like Etoposide, Procarbazine and Cyclophosphamide. Clinically, patients present with a morbilliform maculopapular eruption, papulonodular lesions, erythematous patches, or Sezary-like erythroderma. Histology most often shows a superficial perivascular and lichenoid band-like lymphocytic infiltrate with enlarged, atypical, hyperchromatic lymphoid cells, with irregular nuclear contours and occasional conspicuous nucleoli. CD30 may highlight solitary units and small clusters of enlarged atypical cells in the dermal infiltrate. Epidermotropism is often seen, but there is accompanying spongiosis with dermal edema, interface vacuolar alteration with necrotic keratinocytes, and dermal eosinophils. T-cell lymphomatoid drug reactions may also present with a dense nodular dermal lymphoid infiltrate with cytologic atypia. An admixed histiocytic granulomatous component with giant cells and eosinophils, can be a clue for a pseudolymphomatous process. The nodular infiltrates typically melt following cessation of therapy. Additionally, gene rearrangement studies in T-cell pseudolymphomatoid drug reactions mostly show evidence of polyclonality. Lamotrigine has been implicated in drug-induced hypersensitivity “DRESS” (Drug rash with eosinophilia and systemic symptoms) syndrome. It however, has rarely been associated with CD30+ T-cell pseudolymphomatoid drug reaction, & definitely not with CD30+ T-cell lymphoproliferative neoplasm. Our patient was on antiepileptic medication Lamotrigine, and presented with worsening of her cutaneous symptoms with its dose increment, & persistence of her rash despite drug discontinuation. The histology of the dermal infiltrate with atypia, pleomorphism & CD30 expression, in conjunction with demonstration of T-cell clonality on gene rearrangement studies, is consistent with diagnosis of cutaneous CD30+ T-cell lymphoma, ALCL type. The clinical presentation strongly implicates Lamotrigine as a possible etiologic factor for the lymphoma.

References: 1. Magro CM, Crowson AN, Kovatich AJ, Burns F. Drug-induced reversible lymphoid dyscrasia: a clonal lymphomatoid dermatitis of memory and activated T cells. Hum Pathol 2003;34:119–129. 2. Magro CM, Olson L, Nguyen GH, Feraudy SM. CD30 positive Angiocentric Drug Reactions: Characterization of a Series of 20 Cases. Am J Dermatopathol. 2017 Jul;39(7):508-517

39

Case No: 9b

Localized CD30 and CD4 Positive Cutaneous T-Cell Lymphoma

Care provider: Gage Rensch, MD New Orleans, Louisiana

History: A 60-year-old white female presents with a pruritic scaly papule on the left dorsal hand. The spot grew slowly over a few weeks prior to her appointment. A scaly macule appeared on her left forearm that resolved spontaneously before presentation. No personal history of skin cancer was reported. Her medical history was otherwise noncontributory.

Physical Examination: Skin: Well-circumscribed violaceous papule (6 mm x 7 mm) with broad central dell (hyperkeratotic scale resembling SCC before biopsy) on the dorsal thenar web space of the left hand (see clinical photo). Lymphatics: No lymphadenopathy palpable in the following nodal basins: auricular, cervical, supraclavicular, axillary, supratrochlear or inguinal chains.

Histopathology: Shave Biopsy, Left Radial Dorsal Hand, rule-out SCC Lichenoid and nodular lymphocytic infiltrate of enlarged atypical lymphocytes with irregular nuclear contours. Marked epidermotropism. Immunostains showing T cell population of CD3+ with elevated CD 4:8 ratio. There was staining for CD30 in addition to the CD4 ratio. CD45 was positive. CD20 showed scattered B Cells. Alk-1 was negative. CD1a, S100, CD117, EMA, SOX10 were negative.

Clinical Course: Given the solitary nature of the disease, the patient was scheduled with for wide local excision. No additional workup was ordered as she lacked systemic symptoms, exhibited no lymphadenopathy, and had a stable solitary lesion. The patient will follow up in 2 to 3 months for repeat full body skin examination.

Diagnosis: CD30 and CD4 positive Cutaneous T-Cell Lymphoma (CTCL)

Points of Emphasis: We present this case for discussion of our patient’s differential diagnosis. We favor pagetoid reticulosis (localized) versus primary cutaneous anaplastic large cell lymphoma (C-ALCL) over (LyP). Woringer-Kolopp disease, the localized variant of pagetoid reticulosis, is a very rare form of CTCL. Patients characteristically present with solitary hyperkeratotic papules, nodules, patches or plaques localized to extremities. Histology shows large atypical pagetoid cells with hyperchromatic and cerebriform nuclei singly or in nests with

40 impressive infiltration of the epidermis.1 Immunostaining typically reveals predominance of CD3+ plus either CD4+ or CD8+ T Cells. CD30 is often expressed, as in the case of our patient. The preferred treatment is radiotherapy or surgical excision, although recent cases of successful treatment with topical imiquimod, topical bexarotene and photodynamic therapy have been reported.2-4 C-ALCL and LyP may be conceptualized as opposite ends of a spectrum of primary cutaneous CD30+ lymphoproliferative disorders (C-ALCL at one end and the more indolent LyP at the other with borderline cases in between). C-ALCL presents as a solitary or localized papule, nodule or tumor that may ulcerate. Histology often shows large anaplastic or pleomorphic CD4+ T-cells with abundant cytoplasm and round or irregularly shaped nuclei with prominent nucleoli.5 CD30 should be positive in the majority of tumor cells. Although epidermotropism may be observed, non-epidermotropic infiltrates with cohesive sheets of cells in the dermis are more characteristic.5 Of note, ALK-1 expression is present in the majority of cases of nodal ALCL whereas it is typically absent in C-ALCL. Radiotherapy or surgical excision is the initial treatment in patients presenting with solitary lesions. LyP is a low-grade variant of CTCL characterized by recurrent, disseminated, self-healing papules and nodules. Individual lesions typically disappear in 3 to 12 weeks. Our patient had a second lesion involving the left forearm that resolved prior to her appointment which would be consistent with this diagnosis. However, the biopsied lesion on her hand persisted and progressed over 2 months with no evidence of involution. Numerous histopathologic subtypes exist for LyP. LyP Type B is most consistent with our case and is characterized by epidermotropic pattern with CD4+ and CD30+ T-Cells with pleomorphic, enlarged and cerebriform nuclei. Treatment is often unsatisfactory. For solitary lesions that do not resolve, in the case of our patient, topical steroids, intralesional steroids, and surgical excision are often utilized.6-8 Patients require close follow-up for monitoring for progression of disease. Overall, our patient’s diagnosis is most consistent with C-ALCL versus localized variant of pagetoid reticulosis. In general, localized variants of the diseases discussed may all be treated with surgical excision, and thus, we agreed it was the best course of action in our patient. Close follow-up was scheduled to monitor for disease recurrence or new lesions.

References: 1. Cerroni L, Hodak E, Kempf W, et al., Variants of mycosis fungoides. In: Elder DE, et al.: WHO classification of skin tumours (4th ed). Lyon, 2018, p 231-233, ISBN 978- 92-832-2440-2. 2. Su, O. , Dizman, D. , Onsun, N. , Bahali, A. , Biyik Ozkaya, D. , Tosuner, Z. and Demirkesen, C. (2016), Treatment of localized pagetoid reticulosis with imiquimod: a case report and literature review. J Eur Acad Dermatol Venereol, 30: 324-326.

41

doi:10.1111/jdv.12718 3. Ledet, J. , Groysman, V. and Sami, N. (2016), Woringer–Kolopp (Pagetoid Reticulosis) disease successful response to bexarotene gel. Dermatologic Therapy, 29: 28-31. doi:10.1111/dth.12281 4. Kaufmann, F. , Kettelhack, N. , Hilty, N. and Kempf, W. (2017), Unilesional plantar mycosis fungoides treated with topical photodynamic therapy – case report and review of the literature. J Eur Acad Dermatol Venereol, 31: 1633-1637. doi:10.1111/jdv.14160 5. Massone, C. , El-Shabrawi-Caelen, L. , Kerl, H. and Cerroni, L. (2008), The morphologic spectrum of primary cutaneous anaplastic large T-cell lymphoma: a histopathologic study on 66 biopsy specimens from 47 patients with report of rare variants. Journal of Cutaneous Pathology, 35: 46-53. doi:10.1111/j.1600- 0560.2007.00778.x 6. Martinez-Cabriales, S. A., Walsh, S. , Sade, S. and Shear, N. H. (2019), Lymphomatoid papulosis: an update and review. J Eur Acad Dermatol Venereol. Accepted Author Manuscript. doi:10.1111/jdv.15931 7. Fernández-de-Misa, R et al. (2018), First-line treatment in lymphomatoid papulosis: a retrospective multicentre study. Clin Exp Dermatol, 43: 137-143. doi:10.1111/ced.13256 8. Wieser, I., Won Oh, W., Talpur, R., Duvic, M. (2016), Lymphomatoid papulosis: Treatment response and associated lymphomas in a study of 180 patients. Journal of the American Academy of Dermatology, 74: 59-67. doi: 10.1016/j.jaad.2015.09.013

42

Case No: 10a

CD30 + Large Cell Anaplastic Lymphoma

Care provider: Daniel Witheiler, MD Clay J. Cockerell, MD Dallas, Texas

History: A 72 year old African American woman presented with a slowly growing asymptomatic ulcerated lesion of the left lower leg that had been present for a number of months. The clinical impression was possible squamous cell carcinoma or amelanotic melanoma

Physical Examination: A solitary large, eroded tumor was present of the leg with no other lesions. No lymphadenopathy was noted and there were no lesions elsewhere.

Laboratory Data: N/A

Histopathology: There was a poorly differentiated malignant neoplasm comprised of sheets of atypical cells with superficial erosion. No melanin was seen and there was no definitive epithelial differentiation. There was brisk inflammation with scattered eosinophils. Immunoperoxidase stains were strongly positive for CD30 with negative staining for Sox-10 and cytokeratin. Additional stains were positive for CD3 and negative for CD20.

Diagnosis: CD30 + large cell anaplastic lymphoma

Clinical Course: The patient underwent surgical excision. Work up revealed no evidence of distant disease.

Points of Emphasis: Anaplastic large cell lymphomas (ALCLs) are distinguished from other lymphomas by their anaplastic cytology and constant membrane expression of the CD30 antigen (an activation marker for B or T cells). Striking clinical features include frequent cutaneous and extranodal involvement, young age at presentation, and male predominance. In addition, an association between ALCL and breast implants, particularly those with textured surfaces, has been reported. The primary cutaneous form of ALCL (PC-ALCL) is defined by skin-only involvement without systemic dissemination at presentation. However, extracutaneous involvement occurs in 5-10% of patients, mostly to the draining regional lymph nodes. PC-ALCL is part of the primary cutaneous CD30+ T-cell lymphoproliferative disorders, a wide spectrum of disease, with lymphomatoid papulosis (LyP) at the benign end of the spectrum and PC-ALCL at the malignant end. Borderline lesions lie somewhere in between.

Primary cutaneous anaplastic large cell lymphoma (ALCL) usually manifests as a single or localized cluster of erythematous skin nodules some of which may demonstrate superficial ulcerations. As many as 25% of patients have some degree of spontaneous

43 regression of these lesions. Although most cases present with local involvement, patients may, in rare instances, present with disseminated cutaneous disease, in which case they are at higher risk of developing spread to other organs.

Solitary or localized primary cutaneous lymphoma is best treated with radiation therapy or surgical excision. Both approaches bring about complete remission in more than 95% of cases. Nonetheless, approximately 40% of the cases relapse within a period of a few months to a few years. Treatment with either of these modalities depends on patient preference, site of disease, and local expertise. Combined-modality treatment (surgery followed by radiation therapy) may prolong the time to first relapse. Spontaneous regression is common. It is reported in 40% or more of patients and does not affect overall outcome.

References: Anaplastic Large Cell Lymphoma. Medscape. Retrieved from: https://emedicine.medscape.com/article/208050-overview#a1

44

Case No: 10b

CD30 POSITIVE LYMPHOMA SIMULATING PYODERMA GANGRENOSUM

Care provider: Leisa Hodges, DO Clay J. Cockerell, MD Dallas, Texas

History: A 51 year old female presents with a chief complaint of a “rash” that had been present for years on the left leg. She complained of pruritus and noted that it had been coming and going, leaving brown depressed areas when lesions would heal. She denies having recent infections, new medications, new personal care products and no new household contacts with similar rash. She had been applying over the counter Neosporin ointment to the lesions.

Physical Examination: There were several well-demarcated ulcerations with jagged undermined violaceous borders distributed on the left knee. There were also several hyperpigmented, firm areas of the thigh at sites where lesions had reportedly been present earlier.

Laboratory Data: Bacteria identified in skin by aerobic culture consistent with normal flora

Histopathology: There was a nodular and diffuse infiltrate of lymphoid cells with enlarged, hyperchromatic nuclei. There were areas of crush artifact and focal areas of necrosis. Immunohistochemistry revealed that majority of the infiltrate was composed of CD3 positive T-lymphocytes with a smaller number of CD20 positive B-lymphocytes. A stain for CD30 was strongly positive in many of the larger, atypical lymphoid cells. PAS and Fite stains were negative for microorganisms.

Clinical Course: The patient is currently being evaluated for presence of disease elsewhere

Diagnosis: CD30 Positive Anaplastic Lymphoma Simulating Pyoderma Gangrenosum

Points of Emphasis: The CD30+ primary cutaneous T-cell lymphoproliferative disorders are the second most common subgroup of cutaneous T-cell lymphomas after mycosis fungoides (MF) accounting for approximately 30% of cases. These cutaneous lymphomas have customarily been classified on the basis of their clinical presentation into lymphomatoid papulosis (LyP), primary cutaneous anaplastic large cell lymphoma (pcALCL), systemic ALCL and borderline cases. In recent years, genomic analysis has become important for the diagnosis and clinical management of patients affected by systemic and cutaneous hematologic malignancies. CD30 antigen, originally identified as a cell surface marker of the malignant Hodgkin and Reed-Sternberg (HRS) cells in Hodgkin’s lymphoma by the use of the Ki-1 monoclonal antibody, is a transmembrane glycoprotein member of the tumor necrosis

45 factor (TNF) receptor superfamily. In lymphoid cells, CD30 is an activation marker inducible in vitro by mitogenic signals and viral stimulation, and its expression is detected in a small number of immunoblasts in benign lymphatic tissues.

Many of these lesions may have an indolent course often being present for many years without undergoing systemic spread and may undergo spontaneous regression with periodic exacerbation. In this case, the process has been present for years before an accurate diagnosis was rendered. While there may be many different clinical presentations, the ulcerative nature of these lesions resulted in changes that simulated pyoderma gangrenosum.

References: 1. de Leval L, Gaulard P. CD30+ lymphoproliferative disorders; Haematologica. 2010 Oct; 95(10):1627-30. doi: 10.3324/haematol.2010.029256. 2. Lucia Prieto-Torres, Socorro M. Rodriguez-Pinilla, Arantza Onaindia, Mariano Ara, Luis Requena, Miguel Á. Piris Haematologica February 2019 104: 226-235; Doi:10.3324/haematol.2018.197152 CD30-Positive Primary Cutaneous Lymphoproliferative Disorders: Molecular Alterations And Targeted

46

Case No: 11

METASTATIC RENAL CELL CARCINOMA

Care provider: Etan Marks, DO Clay J. Cockerell, MD Delray Beach, Florida Dallas, Texas

History: A 65 year old male came in for an evaluation of a skin lesion, located on the neck. Unfortunately, the patient was a poor historian and only recently realized the lesion was present. The lesion was bleeding, not healing, enlarging and moderate in severity. The patient had no history of any type of cancer.

Physical Examination: There was a pearly telangiectatic papule approximately 1 centimeter in diameter that was bleeding. Otherwise, there were no apparent worrisome lesions. The general appearance of the patient was well developed and well-nourished and he was alert and oriented x 3 in no acute distress.

Laboratory Data: No pertinent laboratory data.

Histopathology: There was a nodular proliferation of clear and eosinophilic cells. There was also an abundant vascular network with abundant blood cells. The lesion was fairly well circumscribed, but was present at the deep margin. The stains performed showed that the malignant cells were positive for AE1/3, Androgen Receptor, and CD10. The malignant cells were negative for SOX-10, Ep-CAM, EMA, Melan-A, HMB-45, and RCC. This was most consistent with a metastatic renal cell carcinoma.

Clinical Course: The patient was informed of their diagnosis and is scheduled to have a full work-up, but has yet to follow up.

Diagnosis: Metastatic renal cell carcinoma

Points of Emphasis: Patients with renal cell carcinoma (RCC) develop metastasis in approximately 30% of cases. Metastatic RCC of the skin is very rare, and few cases have been reported in the english literature. The most frequent clinical appearance of cutaneous metastases is that of intra- or subcutaneous, round or oval, nodular masses, of firm or elastic consistency, and highly varied color, ranging from normal skin color, bluish red or purplish to dark brown. In the metastatic dermal lesions, the nodular tumor is covered by atrophic epidermis, with tumor cells of vacuolar appearance, clear cytoplasm, a mild lymphocytic inflammatory infiltrate, abundant capillary formation, and some trabecular areas. The tumor cells usually have an atypical nucleus with a prominent nucleolus.

47

The differential diagnoses of cutaneous metastatic renal cell carcinoma is sebaceous carcinoma, a sweat gland tumor, a cutaneous PEComa and melanoma. The tumor cells are usually positive for PAX8, CD10, RCC, and AE1/3.

References: (At least two recent pertinent published references) 1. Mahmoudi, H.R., Kamyab, K. and Daneshpazhooh, M. (2012) Cutaneous Metastasis of Renal Cell Carcinoma: A Case Report. Dermatology Online Journal, 18, 12. 2. Carlesimo M, Abruzzese C, Narcisi A, Arcese A, Cortesi G, Di Russo PP, Fidanza L, Feliziani G, Gargano C, Bartolazzi A, Camplone G. Renal cell carcinoma diagnosed by cutaneous metastasis: a case report. Cutis. 2012 Oct;90(4):196-199.

48

Case No: 12

NEUROENDOCRINE CARCINOMA

Care provider: Helen Kaporis, DO Clay J. Cockerell, MD Dallas, Texas

History: 78 y/o female with a remote history (about 10 years ago) of non-small cell lung cancer status post lobectomy and chemotherapy. She presented to the clinic with a scalp nodule. The pathology of the nodule revealed a neuroendocrine carcinoma.

Physical Examination: She had a Pet-CT scan that showed innumerable pulmonary nodules, hepatic metastasis, osseous metastasis of the ribs and mets to the calvarium; and an MRI that showed multiple brain metastases.

This was felt to be a very aggressive primary small cell carcinoma of the lung.

Laboratory Data: No pertinent lab results shared.

Histopathology: Immunoperoxidase stains were strongly positive for thyroid transcription factor-1 (TTF-1) and insulinoma-1 (INSM1). Stains for cytokeratin and cytokeratin-20 were negative. This suggests that the lesion is metastatic and, given that the patient has history of lung cancer, likely from metastatic small cell carcinoma of the lung.

Clinical Course: The patient passed away on July 30th 2019, before chemo could be started.

Diagnosis: Neuroendocrine Carcinoma

Points of Emphasis: Poorly differentiated adenocarcinoma within dermis and often underlying subcutaneous tissue. Tumor cells are often found in sheets or clusters or as single detached cells and there is variable desmoplasia or sclerosis of dermal collagen.

Thyroid transcription factor, also called thyroid specific enhancer binding protein 38 kDa nuclear protein that regulates transcription activity of thyroid (thyroglobulin, thyroperoxidase, sodium-iodide transport protein, calcitonin and MHC class I), lung (surfactant proteins A, B and C, Clara cell secretory protein) and diencephalon specific genes. Mutations cause pulmonary hypoplasia and neonatal death. Specificity depends on antibody clone used. Interpretation: nuclear stain TTF1 Positive staining is normal in Lung type II pneumocytes and Clara cells; thyroid follicular and parafollicular C cells. Conversely, TTF1 Positive staining seen in disease states such as lung carcinoma: small cell (90%), adenocarcinoma (75%); large cell

49

(40%); squamous cell (5%, Am J Surg Pathol 2001;25:363) Thyroid: hyperplastic and neoplastic thyroid tissue, but less common / lower levels in undifferentiated thyroid carcinomas (Mod Pathol 2000;13:570) Other tumors: small cell carcinomas of lung and various sites (Mod Pathol 2000;13:238; Am J Surg Pathol 2001;25:815); colorectal carcinomas to lung may be positive (Mod Pathol 2005;18:1371), small cell carcinoma of bladder (39%, Hum Pathol 2005;36:718) Negative staining Merkel cell carcinoma of skin, most nonpulmonary non-small cell carcinomas.

References: 1. Pernick, Nat M.D. PathologyOutlines.com Stains TTF1 Topic Completed: 1 June 2005 Revised: 25 April 2019 Copyright: 2003-2018, PathologyOutlines.com, Inc. 2. Tranesh, Ghassan A., M.D. Qu, Hong M.D. Skin nonmelanocytic tumor, other tumors of skin, Metastatic carcinoma to skin Topic Completed: 1 November 2014 Revised: 11 April 2019 Copyright: 2002-2019, PathologyOutlines.com, Inc.

50

Case No: 13a

Primary Cutaneous Marginal Zone B-cell Lymphoma vs. Pseudolymphoma

Care provider: Katharine Saussy, MD Andrea Murina, MD New Orleans, Louisiana

History: A 43-year-old Hispanic gentleman presented to Tulane Dermatology with a reported past medical history of pseudolymphoma of the right lower leg diagnosed by biopsy at an outside hospital. He stated after the lesion was “ruled out” for lymphoma, it was treated with intralesional kenalog and resolved. At his initial visit, the patient presented with a 1 cm, erythematous blanchable papule. No treatment was initiated at the initial visit. Five months later, the patient presented for reevaluation of the lesion as he was reportedly playing tennis and noticed the lesion recurred. He endorsed the lesion was mildly itchy, non-tender, and otherwise asymptomatic.

Physical Examination: 2/2018: Right lower leg with a 1 cm, erythematous, blanchable papule 7/2018: Right lower leg with a 2 cm, brightly erythematous and indurated nodule

Histopathology: A punch biopsy was performed which was read as a low-grade b-cell lymphoma most consistent with primary cutaneous marginal zone b-cell lymphoma. The biopsy was reviewed by Tulane dermatopathology and hematopathology. Skin biopsy showed fragments of skin involved by an atypical lymphoid infiltrate consisting mostly of small to intermediate sized mature lymphocytes with focal germinal center formation also evident on H&E. No diffuse sheets of large atypical lymphoid cells that might suggest a large cell lymphoma were appreciated. No significant plasmacytic component to the infiltrate was seen. No epidermotropism was observed. Cell Phenotype: CD3 and CD20 stains highlighted T-cells and B-cells with B-cells predominating. The B-cells co-expressed CD79a with germinal center B-cells also showing co- expression of CD10 and BCL-6, negative for BCL-2 and a high Ki-67 proliferation index (normal staining pattern). The B-cells outside of the germinal center were negative for CD10 and BCL-6, positive for BCL-2, and had a low Ki-67 proliferation index. A subset of non-germinal center B-cells showed strong expression of CD23. No CD5+, cyclinD1+, or MUM1+ B-cells were appreciated. No definitive kappa or lamda light chain expression by B-cells were seen. Molecular studies for B-cell receptor gene rearrangements were performed. A positive clonal expansion of B-cell population on the IgK gene rearrangement study was found.

Clinical Course: The patient was referred to /oncology for further evaluation. PET-CT was ordered which revealed no pathologically enlarged LN. A bone marrow biopsy was performed which showed mildly hypocellular marrow for age (40%)

51 with trilineage hematopoiesis, a normal myelod:erythroid ratio (2:1), adequate megakaryocytes, and adequate iron stores. No morphologic or immunophenotypic evidence of involvement by lymphoma. It was recommended to have lesion excised. The patient relocated back to Miami where the excision pathology was again read as pseudolymphoma.

Diagnosis: Primary Cutaneous Marginal Zone B-cell Lymphoma (PCMZL)

Points of Emphasis: Pseudolymphoma and primary cutaneous marginal zone b-cell lymphoma (PCMZL) are difficult to distinguish from each other as both have similar clinical, dermatoscopic, and histologic findings. PCMZL is most commonly seen in middle-aged adults, M>F, and is often described as a 2-3 cm, erythematous nodule on the trunk or extremity.1 It’s often considered a low-grade b-cell lymphoma and has a highly favorable 5-year prognosis (>90%). Pseudolymphoma has classically been known to represent a non-malignant, reactive process that often mimics a true lymphoma clinically and histologically, but ultimately behaves as a localized, benign process with the ability to resolve on its own or with minimal localized treatment such as intralesional steroid injections. On physical examination and dermoscopy, both are often described as a solitary (or multiple) pink to erythematous nodule with a polymorphous vascular pattern and evidence of white, circle or streak-like areas.1 In addition, both entities have similar histologic features, mainly a diffuse lymphoid proliferation involving T cells, B cells, or often both. PCMZL reveals a dense lymphoid infiltrate with both CD3+ T cells and CD79a B cells, consisting of small to medium sized cells.3 Studies comparing entities on the pseudolymphoma spectrum, such as pseudolymphoma folliculitis, note a significant increase in staining in PD-1+ T cells as a distinguishing feature possibly found in lymphoid hyperplasias when compared to marginal zone lymphomas.2 PCMZL is noted to often have co-expression of CD34+ cells by mature B cells, light-chain restriction, and a clonal IgH gene rearranagement.3 However, there appears to be no clear diagnostic criteria. Given the challenge to discern between the two entities, with differing opinions on pathology findings, we present this case for discussion of the pathology results for this patient. Our intent for discussion is to help clarify the distinguishing features of pseudolymphoma compared to primary cutaneous marginal zone b-cell lymphoma.

References: 1. Bombonato C, Pampena R, Lallas A, et al. Dermoscopy of Lymphomas and Pseudolymphomas. Dermatol Clin. 2018 Oct;36(4):377-388. 2. Goyal A, Moore B, Gimbel D, et al. PD-1, S-100 and CD1a expression in pseudolymphomatous folliculitis, primary cutaneous marginal zone B-cell lymphoma (MALT lymphoma) and cutaneous lymphoid hyperplasia. J Cutan Pathol. 2015 Jan;42(1):6-15. 3. Lan TT, Brown NA, and Hristov AC. Controversies and considerations in the diagnosis of primary cutaneous CD4+ small/medium T-cell lymphoma. Arch Pathol. Lab Med. 2014 Oct;138(10):1307-18.

52

Case No: 13b

Cutaneous T-cell Lymphoma and Concurrent Primary Cutaneous Marginal Zone B-Cell Lymphoma

Care provider: Garrett Vick, MD Lucy Chao, MD Abida Kadi, MD New Orleans, Louisiana

History: This is a 61-year-old Vietnamese man who reported a past medical history of cutaneous T-cell lymphoma (CTCL), diagnosed in 2008, for which he received systemic chemotherapy in 2015. He was unable to verify the agents administered at that time, but was noted to have a medical port in place through which he reported having received treatment. He then went untreated for 3 years before presenting in October of 2018 for a chronic skin eruption concerning for potential recurrent CTCL. Skin biopsy showed nodular dermal lymphocytic infiltrate consisting of mixed atypical B-lymphocytes, T- cells, and plasma cells around foci of germinal centers. The cell phenotype was noted to be CD20+/CD79a+/CD138+/ with lambda light chain restriction. Peripheral blood flow cytometry showed no increase in blasts and bone marrow biopsy in November of 2018 showed mild hypercellularity with a minute population (2%) of CD4+, CD7- T-cells. These cells were thought to be reactive, but were similar to those seen on peripheral flow (see below), thus not completely ruling out the possibility of CTCL. The patient was diagnosed with primary cutaneous marginal zone B-cell lymphoma (PCMZBCL). He was prescribed rituximab, but experienced a severe allergic reaction during the initial administration in December of 2018 which required desensitization after which he completed 3 additional doses of rituximab before being continued on a maintenance regimen of rituximab every 2 months which resulted in improvement noted by hematology/oncology.

In April 2019, the patient was noted to have a skin eruption that was not responding to rituximab. He was referred for dermatologic evaluation and repeat skin biopsies were performed. These showed chronic spongiotic dermatitis that did not have significant atypia noted. However,gene rearrangement studies demonstrated positivity for T-cell receptors gamma and beta as well as detection of IG heavy chain, but negative for IG kappa light chain. Given concern for reemergence of previousmCTCL, he underwent a PET/CT performed June of 2019 which showed small areas of increased metabolic activity involving the skin of the left posterior thorax. He was subsequently started on bexarotene 150mg daily July 2, 2019, but then was hospitalized July 24, 2019 with MSSA bacteremia for which his medical port was removed and he was treated with intravenous nafcillin until August 22, 2019. During hospitalization for bacteremia, he was also noted to have elevated liver enzymes leading to discovery of serology suggestive of latent hepatitis B (positive core antibody, but negative surface antigen and antibody). While undergoing MRCP for recommended liver biopsy, he was incidentally noted to have a 3cm renal mass concerning for renal cell carcinoma (RCC) which is pending

53 further work up by at this time. Liver biopsy was not performed. He was started on prophylactic entecavir to be continued for 6-12 months after stopping rituximab. His last dose of rituximab was May of 2019 and per hematology/oncology he was planned to have 2 more doses prior to discovery of potential RCC.

Physical Examination: VS: 147.8 lbs General: Well developed, well nourished, in no acute distress HEENT: No oral or conjunctival lesions noted on inspection Skin of the scalp, face, ears, neck, chest, abdomen, back, both upper extremities, and both lower extremities were examined and were remarkable for: Ears: Lichenoid plaques of bilateral helices. Neck: Scattered erythematous scaly patches and thin plaques Back: Scattered erythematous scaly patches and thin plaques on the lower back. Right arm: Scattered erythematous scaly patches on the upper arm. Left arm: Scattered erythematous scaly patches on the upper arm. Right leg: Scattered erythematous scaly patches on the upper thigh. Left leg: Scattered erythematous scaly patches on the upper thigh.

Laboratory Data: 10/2018 Peripheral blood flow cytometry: No increased blasts found. No increased or monoclonal B-cell population is found. CD4+ T-cells within normal range, but significant loss of CD7 (60%); cannot exclude rare circulating CTCL cells.

HTLV1/2: negative

05/2019 CBC remarkable for mild neutropenia of 49% and eosinophilia of 15%.

07/2019 Hepatitis B core antibody positive; surface antigen and surface antibody negative.

09/2019 Sezary cell panel with few unusual mature small to intermediate sized lymphocytes with folded nuclei, dark chromatin, and modest amount of agranular cytoplasm. These few unusual cells may possibly represent Sezary cells. Flow cytometry noted to have few CD4+, CD7-, CD25+, CD30+ T-cells suggestive of Sezary cells.

Histopathology: 10/2018 Skin, superior left back, biopsy: Atypical lymphocyte infiltrate with follicular mucinosis, compatible with cutaneous marginal zone B-cell lymphoma Skin, inferior left back/superior right inner thigh/inferior right inner thigh, biopsy: Primary cutaneous marginal zone B-cell lymphoma

54

11/2018 Bone marrow, aspirate smears, biopsy and aspirate clot: Mildly hypercellular bone marrow (60%) for age with trilineage hematopoiesis and mild hyperplasia and atypia in megakaryocytic lineage. Minute population of CD4+, CD7- T-cells (2% of total events), and a minute population of CD8+, CD7- T-cells (1% of total events) detected by flow cytometry study.

4/2019 Skin, left inferior back, biopsy: Chronic spongiotic dermatitis with superficial dermal fibrosis, capillary hyperplasia and plasmacytosis; no significant lymphocytic atypia seen. T-cell receptor gamma gene rearrangement positive with clonal rearrangement detected. IG Kappa light chain gene rearrangement not detected.

Skin, left superior back, biopsy: Chronic spongiotic dermatitis with superficial dermal fibrosis and capillary hyperplasia; mildly atypical superficial and deep dermal perivascular and interstitial lymphohistiocytic infiltrate with eosinophils and plasma cells. T-cell receptor beta chain gene rearrangement positive with clonal rearrangement detected. IG Heavy chain gene rearrangement positive indicating clonal b cell population present.

Clinical Course: Most recently, he was seen by dermatology on September 10, 2019 for worsening of skin involvement with thickening of firm plaques to skin. A Sezary panel performed after that visit demonstrated a few circulating Sezary cells. Thus, the patient was restarted on topical triamcinolone 0.1% cream and bexarotene 150mg daily which he had not be taking during recovery from acute bacteremia. He also continues on entecavir for HBV prophylaxis and is pending further diagnostic work up for suspected renal cell carcinoma.

Diagnosis: Cutaneous T-cell Lymphoma and Concurrent Primary Cutaneous Marginal Zone B-Cell Lymphoma

Points of Emphasis: Given the unusual nature of this patient’s condition, we present this case for discussion in efforts to clarify the patient’s current diagnosis. Our hope is that clarification will guide appropriate next steps in management and to help avoid harm such as proceeding with therapy that may potentially worsen the status of his underlying disease. We are unsure as to which of the following scenarios is most accurate or if an entirely different alternative explanation exists. The first possibility is that the patient appears to have concurrent discordant lymphomas with two distinct lymphoproliferative disorders (CTCL and PCMZBCL) occurring sequentially or perhaps even as a composite lymphoma with multiple distinct lymphoma subtypes of which a similar case was reported in 2006 where an elderly woman was noted to have findings consistent with three simultaneous lymphomas including primary cutaneous marginal zone B-cell lymphoma, nodal Epstein–Barr virus associated classic Hodgkin’s lymphoma (B cell type), and peripheral T-cell lymphoma coexisting in the skin and cervical lymph node (Nishioka, Steinhoff). Interestingly, and in a similar fashion to

55 the case above, this patient was noted to have had a history of nodal T-cell lymphoma that had been diagnosed and successfully treated with 13 cycles of Belcomycin, Etoposide, Adriamycin, Cyclophosphamide Oncovin, Procarbazine, and Prednisone followed by two cycles of Cyclophosphamide, Oncovin, Procarbazine, and Prednisone 30 years prior to her subsequent presentation with multiple concurrent lymphomas. Thus, a sequence of events could be considered where the treatment for one original lymphoma may have predisposed to the development of another single or multiple lymphoproliferative disorders. The second explanation is that our patient may have been experiencing manifestations of one cutaneous lymphoma (T-cell or B-cell) throughout the above history with periods in which some of the biopsies demonstrated an additional confounding monoclonal B- cell or T-cell rich component respectively. Such a case was reported by Gallardo et al in 2006 where an elderly woman was found to have primary cutaneous marginal zone B- cell lymphoma with a prominent T-cell component and an aberrant dual T-cell and B-cell genotype (Gallardo). A review of the literature reveals that there have been a few rare cases where marginal zone B-cell lymphomas have even mimicked cutaneous T-cell lymphoma with lichenoid infiltrates and epidermotropism as notable features (Lee, BA). In such cases, immunohistochemistry is required to verify the diagnosis of marginal zone lymphoma which demonstrates CD20+, bcl-2+, bcl-6−, CD10− and CD5− staining as well as light chain restriction some features of which were noted on biopsy in our patient in 2018. That biopsy also showed some T-cell infiltrate that rimmed around neoplastic B-cells and in some areas showed infiltrate of mainly T-cells, however biopsy features were overall felt to be consistent with PCMZBCL at that time. Thus, in summary, we are unsure of the exact explanation for the patient’s condition. Despite all of the above possibilities, it is also possible that what we may be noting is the presence of atypical T-cells that were circulating peripherally and subsequently found incidentally on biopsy as bystanders that homed to the skin in the setting of his original PCMZBCL. We hope that further discussion of this case will provide constructive critical thought that may help clarify and point to the most accurate diagnosis.

References: 1. Nishioka A, Ureshino H, Ando T, Kizuka H, Kusaba K, Sano H, Itamura H, Kubota Y, Kojima K, Ohshima K, Kimura S. Three coexisting lymphomas in a single patient: composite lymphoma derived from a common germinal center Bcell precursor and unrelated discordant lymphoma. Int J Hematol. 2018 Jun; 107(6):703-708. doi: 10.1007/s12185-017-2370-6 2. Steinhoff M, Assaf C, Anagnostopoulos I, Geilen CC, Stein H, Hummel M. Three coexisting lymphomas in one patient: genetically related or only a coincidence?. J Clin Pathol. 2006 Dec; 59(12): 1312-5 3. Gallardo F, Pujol RM, Bellosillo B, Ferrer D, García M, Barranco C, Planagumá M, Serrano S. Primary cutaneous B-cell lymphoma (marginal zone) with prominent T-cell component and aberrant dual (T and B) genotype; diagnostic usefulness of laser- capture microdissection. Br J Dermatol. 2006 Jan; 154(1): 162-6.

56

4. Lee BA, Jacobson M, Seidel G. Epidermotropic marginal zone lymphoma simulating mycosis fungoides (†). J Cutan Pathol. 2013 Jun; 40(6): 569-72. doi: 10.1111/cup.12114.

57

Case No: 14

Rare Diffuse Large B-cell Lymphoma, Intravascular Type

Care provider: Lindsey Bush Derrick Moody, MD Dipti Anand, MD Atlanta, Georgia

History: A 58-year-old female presented with a non-tender, non-pruritic rash involving the medial aspect of her right thigh. The affected area had progressively enlarged over the course of 3 months. Patient denied any recent history of fever, myalgia, headache, dizziness, weakness, or weight loss. No significant past medical history.

Physical Examination: Exam revealed an ill-defined, poorly circumscribed, hyperpigmented, non-scaly patch located on the anteromedial aspect of patient’s right thigh.

Histopathology: Punch biopsy from the right thigh showed deep intravascular proliferation of atypical enlarged hyperchromatic lymphoid cells with pleomorphism, irregular nuclear contours, and some mitotic figures. Intravascular thrombi with nuclear debris were also present. The adjacent dermis showed a mild perivascular lymphohistiocytic infiltrate. On IHC, the intravascular lymphocytes were positive for CD20, CD5, and bcl-2; and negative for CD3, SOX-10, pan-CK, and p40. Additionally, EBER in situ hybridization was negative for Epstein-Barr Virus.

Clinical Course: The patient’s subsequent staging for systemic lymphoma was negative. She presents with no neurologic dysfunction and no other cutaneous lesions. Further work up including complete blood count, comprehensive metabolic panel, and liver function tests has been normal.

Diagnosis: Diffuse Large B-cell Lymphoma, Intravascular Type

Points of Emphasis: We present a case of Intravascular Large B-Cell Lymphoma (IVLBCL). The clinical presentation is most consistent with localized cutaneous subtype, of Western variant of IVLBCL. Intravascular Large B-Cell Lymphoma, also called intravascular lymphomatosis or malignant angioendotheliomatosis, is a very rare aggressive form of lymphoma that is characterized by selective growth of neoplastic B- lymphocytes within blood vessels. It is a disease of the elderly, presenting with an equal sex predilection, and is usually widespread at presentation. Most common sites of involvement are the CNS and the skin. Cutaneous lesions present as erythematous, violaceous, ill-defined, infiltrating patches and plaques with telangiectasia, most commonly on the arms, thighs, trunk, and face. The rash can also have a livedo reticularis-like pattern with purpura, ecchymosis and gangrene, or thrombophlebitis-like features, or peau d’orange’-like color change. The lung, kidney, GI tract, genitourinary system, endocrine glands, liver, spleen, and BM may also be involved, but lymph nodes

58 are typically spared. Type B symptoms, pancytopenia, hypoxemia and hypoalbuminemia are common. Patients often have raised LDH, β2-microglobulin, ESR & serum ferritin levels. The disease presents with two variants, Western and Asian. The Western variant affects mainly the skin and CNS. The Asian variant, most frequent in Japan, often presents with multiorgan failure, liver and spleen involvement, pancytopenia, and typically a hemophagocytic syndrome. Bone marrow involvement is more common, and skin involvement is less frequent in this variant. The classic form of the disease has an aggressive course and dismal outcome, with 3-year overall survival rate of 22%.

Histology is subtle showing dermal vascular telangiectasia and perivascular inflammation. Careful inspection reveals intravascular proliferation of enlarged, atypical, hyperchromatic lymphocytes which are CD20+ B-cells. The etiology of IVLBCL is unknown, although defects in expression of the homing receptors CD29 (β1 integrin) and CD54 (ICAM-1) is postulated to account for the intravascular location of tumor cells. Our patient presents with Western variant (localized cutaneous subtype) of IVLBCL. This subtype occurs almost exclusively in females. Patients tend to be younger, with a median age of 59, and usually have normal leukocyte and platelet counts, a lower incidence of Type B symptoms, and good performance scores. Prognosis is more favorable in the cutaneous variant, particularly if there is only a solitary lesion. In summary, IVLBCL is a rare, aggressive, often fatal lymphoma frequently involving the skin. It is important for both clinicians and pathologists to be aware of this entity, as the symptoms can be diverse and non-specific, resulting in delayed diagnosis. Intravascular location of malignant lymphocytes can be subtle, and a high index of suspicion is needed when evaluating cutaneous biopsies of erythematous patches/plaques showing non-specific vascular telangiectasia, proliferation and inflammation. Prompt histologic diagnosis is imperative in instituting early management.

References: 1. Nakamura S, Ponzoni M, Campo E. Intravascular large B-cell lymphoma. In: Swerdlow SH, Campo E, Harris NL, et al., eds. WHO Classification of Tumours of Haematopoietic and Lymphoid Tissues. Lyon: IARC Press, 2008: 252–253. 2. Yegappan S, Coupland R, Arber DA, et al. Angiotropic lymphoma: an immunophenotypically and clinically heterogeneous lymphoma. Mod Pathol 2001;14:1147–1156.

59

Case No: 15

Atypical lymphocytic lobular panniculitis

Care provider: Aditya Sood M1 Dustin Portela DO Dipti Anand MD Boise, Idaho Smyrna, Georgia History: 48 year old Caucasian male with no significant past medical history, presented with acute onset of non-tender, non-pruritic, erythematous patches on each leg. He denied constitutional symptoms including fever, night sweats, weight loss, and myalgia.

Physical Examination: There were subtle erythematous patches with palpable underlying nodules, symmetrically distributed on bilateral legs. Examination revealed no organomegaly or lymphadenopathy. Clinical differential diagnoses included erythema nodosum and subcutaneous panniculitis-like T cell lymphoma (SCPLTCL).

Histopathology: A punch biopsy showed a lobulocentric panniculitis with small to intermediate-sized lymphocytes rimming the adipocytes. The lymphocytes showed hyperchromatic nuclei with slight irregular nuclear contours and inconspicuous nucleoli. However, there was no marked cytologic atypia, mitotic activity, fat necrosis, vasculitis, or foci of erythrophagocytosis. No significant epidermal or dermal changes were seen. The histological differential diagnoses included lupus panniculitis, subcutaneous panniculitis-like T-cell lymphoma, primary cutaneous gamma/delta T-cell lymphoma, nasal type extranodal NK/T-cell lymphoma, and atypical lymphocytic lobular panniculitis. Immmunophenotypically, the infiltrate was composed of CD3+ CD5+ CD4+ T-cells, showing weaker expression of CD7. Few scattered CD8+ T-cells were present. Additional work-up showed absence of CD56, Gamma M1 and TIA-1 expression. EBER in situ hybridization for Epstein Barr mRNA was also negative. There was evidence of T-cell clonality, with molecular studies for both T-cell receptors beta and gamma gene rearrangements being positive.

Diagnosis: Atypical Lymphocytic Lobular Panniculitis (ALLP)

Clinical Course: Patient’s serologic work-up showed positive Antinuclear antibodies at a low titer (<1:80). Other laboratory studies including ds-DNA, Lactate dehydrogenase (LDH), complete blood count (CBC), and serum and urine electrophoresis were normal. The patient received intramuscular triamcinolone, and was started on Hydroxychloroquine. He is being closely followed, and a repeat biopsy performed from the left leg, ~5 months later, showed similar lobular panniculitis with no significant lymphocytic atypia. He remains stable with development of a few new lesions confined to his legs.

Points of Emphasis: Atypical lymphocytic lobular panniculitis (ALLP) is a clonal

60 cutaneous lymphoid dyscrasia, exhibiting overlapping features of lupus panniculitis and subcutaneous panniculitis-like T-cell lymphoma (SCPLTCL). The lesions of lupus profundus, ALLP, and SCPLTCL, form a continuum morphologic and biologic spectrum. While both lupus profundus and SCPLTCL show a predilection for middle-elderly aged females, ALLP presents equally in males and females, primarily in young and middle- aged adults. Patients present with chronic recurrent subcutaneous nodules, mostly involving the extremities. While ~50% patients with SCPLTCL present with Type B symptoms, constitutional symptoms are typically absent in patients with ALLP. Unlike the lesions of lupus profundus or SCPLTCL, which do not regress spontaneously, ALLP has a relapsing and remitting course, with a low risk of progression to lymphoma. Histology of lupus profundus, ALLP and SCPLTCL, shows a lobulocentric infiltrate. Absence of hyalinized fat necrosis and dermal lymphoid aggregates, in conjunction with evidence of T-cell clonality, distinguish ALLP from lupus panniculitis. The lobular lymphocytic infiltrate in ALLP lacks the cytologic atypia and high proliferative index seen in SCPLTCL. Absence of fat necrosis and foci of erythrophagocytosis, along with paucity of CD8+ T-cells, and lack of immunophenotypic expression of cytotoxic T-cell markers like granzyme-B, TIA-1 and perforin, help to distinguish ALLP from SCPLTCL. Patients with ALLP need to be closely followed since; an evolving lymphoma cannot be entirely excluded. Repeat biopsy and laboratory studies, with changes in clinical symptoms, lesion morphology, distribution, and progression, and therapy responsiveness, are recommended. The goal of the treatment is to halt the progression of the disease to lymphoma. While corticosteroids and non-steroidal anti-inflammatory agents can be used for pain, oral retinoid therapy has been effective for persistent lesions. In summary, it is important for both clinicians and pathologists to be aware of ALLP, which forms the indeterminate and intermediate spectrum of lobular panniculitis. Patients who present with overlapping features of both lupus profundus and SCPLTCL, but yet lack diagnostic features of either, may be categorized into a separate disease entity, known as ‘Atypical Lymphocytic Lobular Panniculitis’.

References: 1. Magro CM, Schaefer JT et al. Atypical lymphocytic lobular panniculitis: a clonal subcutaneous T-cell dyscrasia. J Cutan Pathol 2008: 35: 947–954 2. Jaffe ES, Gaulard P, Ralfkiaer E, et al. Subcutaneous panniculitis like T-cell lymphoma. In: Swerdlow SH, Campo E, Harris NL,et al., eds. WHO Classification of Tumours of Haematopoietic and Lymphoid Tissues. Lyon: IARC Press, 2008. 294– 295. 3. Magro CM, Crowson AN, Kovatich AJ, Burns F. Lupus profundus, indeterminate lymphocytic lobular panniculitis and subcutaneous T-cell lymphoma: a spectrum of subcuticular T-cell lymphoid dyscrasia. J Cutan Pathol 2001;28:235–247.

61

Case No: 16

Steatocystoma Multiplex and Neurofibromatosis

Care provider: Adeline Kikam McAllen, Texas

History: A 15-year-old Hispanic male immigrant presented with a chief complaint of acne to the trunk and inner thighs oozing thick oily yellowish discharge when squeezed for over two years. He states bumps are increasing in number and while not painful are occasionally pruritic He also complains of generalized dark moles all over his body increasing in size and number from when he was much younger. The patient has a history of Attention Deficit Disorder (ADD) and other unspecified cognitive developmental delays. His mother says she had been advised to have routine evaluations by a neurologist in Mexico but was not told why. MRIs performed in Mexico were unremarkable. He wears prescription lenses but mother does not recall any identification of lisch nodules or optic glioma on routine eye exams. The mother denied any history of neurofibromatosis in the family or any other heritable skin condition. She denied any prior diagnose of NF1 prior to the dermatology visit in her son.

Physical Examination: Physical examination revealed multiple well defined, firm round to oval, smooth, soft movable, yellow to flesh colored papules with no central punctum to the bilateral lower chest, abdomen and inner thigh. Further examination also revealed multiple hyperpigmented macules (Café-au-lait spots) greater than 1.5 cm, axillary freckling and several round soft subcutaneous tumors to the anterior and posterior trunk and thighs. Fig:1&2: Smooth well defined round to oval dermal cysts with no central punctum of 2-5mm in diameter to the anterior lower chest and abdomen. Fig3: Café- au- lait macules to the left lateral flank.

Laboratory Data: None

Histopathology: Fig 4a &b: Mid dermal cyst with wall consisting of ruggated squamous epithelium with a wrinkled crenulated eosinophilic refractile cuticle of keratin instead of a granular layer. Sebaceous gland open into the cyst. The cyst cavity shows keratinous debris. H&E. ×10. Fig 5b: Well demarcated nodule in the deep dermis with proliferation of spindled cells, wavy nuclei, fibroblasts and mast cells interposed between delicate fibrillar collagen diagnostic of a cutaneous neurofibroma. H&E x10 & 20x

Diagnosis: Clinical exam showing three of seven criteria for NF1 along with histopathologic analysis confirmatory for the diagnoses of NF1. Multiple circumscribed papules with oily discharge and a biopsy of one of the papules confirmatory for Steatocystoma Multiplex. Clinical Course: Patient was sent to a pediatric referral center for better collaborative and interdisciplinary management including Neurofibromatosis genetic phenotyping. Steatocystoma treated with observation Points of Emphasis: This case highlights a rare and unique presentation of both

62 conditions in a pediatric patient not previously reported in the literature. Steatocystoma Multiplex and Neurofibromatosis 1 are two separate disease entities with some overlap in clinical presentations between the cutaneous neurofibromas of NF1 and the dermal cysts of SMX. Common to both diseases are mutations on different locations of chromosome 17 with SMX resulting in structural disruptions of keratin production and NF1resulting in dysregulation of tumor suppression ability.

References: 1.Sharma A, Agrawal S, Dhurat R, Shukla D, Vishwanath T. An Unusual Case of Facial Steatocystoma Multiplex : A Clinicopathologic and Dermoscopic Report. Dermatopathology. 2018;5(2):58-63. doi:10.1159/000488584 2. Neurofibromatosis. Conference statement. National Institutes of Health Consensus Development Conference. Arch Neurol. 1988;45(5):575-578.

63

Case No: 17

Dermal fibromatous hamartoma most consistent with medallion-like dermal dendrocytic hamartoma

Care provider: Leisa Hodges, DO Clay J. Cockerell, MD Dallas, Texas

History: A 10 year old Hispanic boy presented with a firm reddish area of the back of the neck that had become noticeable over the last 2 years. It had reportedly been growing in size but was asymptomatic. Clinical impression was possible cyst.

Physical Examination: Solitary firm circumscribed plaque-like area of the posterior neck

Laboratory Data: N/A

Histopathology: Spindle cells in the dermis and upper subcutis arranged in fascicles and some in whorls

Diagnosis: Dermal fibromatous hamartoma most consistent with medallion-like dermal dendrocytic hamartoma

Clinical Course: The patient has been followed. No additional surgery has been performed.

Points of Emphasis: Pediatric spindle cell tumors encompass a large number of different entities. In this case, in the context of the clinical appearance, this most likely represents so-called medallion-like dermal dendrocyte hamartoma. MLDDH is a newly described entity, first reported by Rodriguez-Jurado et al in 2004 in three female children. The lesions were triangular, round, or oval, red-brown, slightly atrophic, and several centimeters in diameter. Histology demonstrated epidermal atrophy and a spindle-cell proliferation in the dermis that stained positively for factor XIIIa and CD34. Mast cells were increased. Two years later, Martin et al reported a similar congenital lesion on the scapula of a 2-month-old female. In 2007, Shaw et al3 reported the fifth case of MLDDH on the breast of a 16-year-old female. The clinical and histopathologic features were identical to the other reported cases with the exception that the spindle cells stained positively for CD34, but not for factor XIIIa. The differential diagnosis includes fibrous hamartoma of infancy which is another spindle cell neoplasm. There are a number of variants dependent on the relative proportion of fat, mature fibrous tissue and primitive mesenchymal elements. Predominantly fatty tumors should be distinguished from lipofibromatosis, the ‘lipofibromatosis-like neural tumor’ and maturing lipoblastoma. Lipofibromatosis usually involves the distal extremities, unlike fibrous hamartoma of infancy, and consists of an

64 infiltrative admixture of mature fat, lipoblast-like cells and short fascicles of bland, fibroblastic cells. The ‘lipofibromatosis-like neural tumor’is characterized by occurrence in the superficial soft tissues of very young children, an infiltrative growth pattern, variable cytological atypia, S100 protein and CD34 expression, and rearrangements of the NTRK1 gene. For chiefly fibroblastic fibrous hamartoma of infancy, the differential diagnosis includes desmoid-type fibromatosis, myofibroma, and calcifying aponeurotic fibroma. Desmoid-type fibromatoses are rare in very young children, and consist of a compact proliferation of ‘active-appearing’, bland myofibroblastic cells arranged in long, sweeping fascicles, surrounding a thin-walled, dilated vasculature. A significant percentage of desmoid-type fibromatoses show aberrant nuclear accumulation of beta- catenin protein and mutations of the CTNNB1 gene; beta-catenin nuclear accumulation is not seen in fibrous hamartoma of infancy. Myofibromas show a distinctive biphasic pattern of growth, with (generally) peripheral zones consisting of hyalinized and myoid- appearing spindled cells and more cellular primitive cells, sometimes with necrosis and/or calcification towards the center of the lesion. Calcifying aponeurotic fibromas contain distinctive calcified, vaguely chondroid-appearing nodules surrounded by epithelioid cells, often with associated osteoclast-like giant cells, in addition to infiltrative fascicles of fibroblastic cells. It has been recently shown that calcifying aponeurotic fibromas are characterized genetically by an FN1–EGF fusion gene. Fibrous hamartoma of infancy showing a predominance of the primitive mesenchymal component and/or prominent hyalinization with cracking artifact are most likely to be confused with giant cell fibroblastoma, a differential diagnosis discussed above. Finally, those exceptionally rare fibrous hamartoma of infancy that contain sarcomatous areas must be distinguished from other pediatric sarcomas, in particular infantile fibrosarcoma and spindle cell rhabdomyosarcoma. Ancillary testing for ETV6 rearrangements, characteristic of infantile fibrosarcoma and immunohistochemistry for desmin, myogenin and MyoD1 (expressed by spindle-cell rhabdomyosarcoma but not fibrous hamartoma of infancy) should facilitate these distinctions.

References:

1. Ducharme, E., Baribault, K., Husain S, and Engler, D. Medallion-like dermal dendrocyte hamartoma in a 36-year-old male. JAAD 59:1:169-172. 2. Al-Ibraheemi A, Martinez A, Weiss SW. Fibrous hamartoma of infancy: a clinicopathologic study of 145 cases, including 2 with sarcomatous features. Mod Pathol 2017 Apr;30(4):474-485.

65

Case No: 18

Degos Disease

Care provider: Aditya Sood M1 David Schoenfeld MD Dipti Anand MD Carrollton, Georgia Smyrna, Georgia

History: 80 year old Caucasian male with no significant past medical history, presented in May 2018 with multiple, crusted and scaly, juicy pink papules, involving his chest & abdomen. The patient’s lesions gradually progressed over a period of 1 year to have atrophic, umbilicated centers, and spread to involve his neck, upper arms and thigh.

Physical exam: Initial physical examination in May 2018, showed multiple (30-40), crusted scaly papules, on chest & abdomen. The clinical differential diagnoses included Grover’s disease, arthropod assault, pityriasis lichenoides chronica, and lymphomatoid papulosis. In June 2019, he had multiple, pink and yellow-gray papules, up to 5 mm in diameter, involving his trunk, neck, upper arms and thighs. The lesions now had developed central atrophy, with a depressed, white, porcelain-like appearance and a fine scale.

Histopathology: Initial shave biopsy of a right chest papule in May 2018, showed epidermal atrophy with hyperkeratosis, focal interface vacuolar alteration, with a patchy lichenoid, and superficial perivascular lymphocytic infiltrate. There was a wedge-shaped are of dermal sclerosis, with pale collagen, and marked interstitial dermal mucin deposition. Repeat shave biopsy from the left abdomen in June 2019, showed similar features with an acellular, sclerotic dermis with mucin, and epidermal atrophy. Additionally, perivascular hyalinization & lymphocytic infiltrate, adjacent to the sclerotic area were seen. The histological differential diagnoses included papular lupus erythematosus and Degos’ disease. Presence of dermal sclerosis and perivascular hyalinization, in conjunction with the clinical presentation, favored Degos disease over lupus erythematosus.

Clinical Course: The patient denied headache, tingling, numbness, weakness, abdominal pain, and joint pain, blood in urine or stool, or weight loss. The neurologic examination did not reveal muscle weakness or sensory loss. No abnormalities were noted on a colonoscopy. Serologic work-up for antinuclear antibodies was negative. Currently, the patient’s cutaneous lesions remain stable. He presents with no intestinal or neurologic involvement, and no manifestations of Lupus erythematosus.

Diagnosis: Degos Disease (Malignant Atrophic Papulosis)

Points of Emphasis: Degos Disease, also known as “malignant atrophic papulosis” is

66 a rare, lethal intestinocutaneous multisystem syndrome of unknown etiology. The disease shows a male predominance (3:1), with presentation most commonly in the 3rd decade of life; however, a wide age range at diagnosis (infancy – 67) has been noted. Familial involvement and presentation during pregnancy has been reported in some cases. The crops of scaly papular cutaneous lesions of Degos disease, with atrophic “porcelain-white” centers are distinctive. However, similar lesions can be a manifestation of other diseases including systemic lupus erythematosus, systemic sclerosis, dermatomyositis, rheumatoid arthritis, and Crohn’s disease. Histology, classically shows cutaneous infarction, manifested as a “wedge-shaped” area of pale, acellular, sclerotic dermis, with mucin deposition and epidermal atrophy. Serial tissue sectioning, may demonstrate a thrombosed vessel at the apex of the lesion. However, hyalinized vessels adjacent and deep to the infarct are often seen. Many debate that Degos disease represents a reaction pattern mainly seen in lupus erythematosus, and is not a specific disease per se. The proposed pathogenetic mechanism is vascular endothelial swelling and proliferation, with alterations of fibrinolysis and platelet function, resulting in secondary thrombosis and ultimately infarction. Degos disease has a high mortality rate and a poor prognosis. Besides cutaneous manifestation, the two most common internal organs involved are intestines, and peripheral and central nervous system. Small bowel involvement with intestinal infarction, perforation & peritonitis, is a common complication, and can be the first presenting manifestation of the disease process. Intestinal disease may precede the cutaneous features, or develop many years after an initial cutaneous presentation. Neurologic involvement with hemi- and quadriplegias, sensory losses, & cranial nerve lesions may dominate the clinical picture. The clinical presentation and disease course in our patient, with cutaneous lesions, and no evidence of intestinal or neurologic involvement, suggests a benign outcome. However, close follow-up for disease evolution and complications is recommended.

References: 1. Calonje, E. (2012). McKee's pathology of the skin: With clinical correlations. Edinburgh: Elsevier/Saunders, 697-698 2. Huang, Y.-C., Wang, J.-D., Lee, F.-Y., & Fu, L.-S. (2018). Pediatric Malignant Atrophic Papulosis. , 141(Supplement 5). doi: 10.1542/peds. 2016-4206 3. Rice AS, Zedek D. Malignant Atrophic Papulosis (Degos Disease) [Updated 2019 Jul 16]. In: StatPearls [Internet]. Treasure Island (FL): StatPearls Publishing; 2019

67

Case No: 19

Dermatomyositis, most consistent with MDA-5 variant

Care provider: Natalie Wright, MD Elba Rubianes, MD Clay J. Cockerell, MD Dallas, Texas

History: An 18 year old Caucasian woman presented with a 6 month history of painful erosions of the hands and feet causing her difficulty in daily activities. She also began developing weakness and difficulty walking and combing her hair.

Physical Examination: Eroded areas of palms and soles with crusted erosions of scattered areas of the trunk and extremities. Light pink erythematous macules of face, elbows and dorsal surfaces of both hands accentuated on the knuckles.

Laboratory Data: Serum aldolase elevated; MRI imaging study confirmed myositis; MDA-5 antibody titer from commercial laboratory negative

Histopathology: Initial biopsy 8/21/19 from ulcerated area revealed erosion and ulceration with sparse infiltrate with no vasculitis. Direct IF negative. Repeat biopsy from dorsal and on 9/11/19 revealed dermatomyositis.

Diagnosis: Dermatomyositis, most consistent with MDA-5 variant

Clinical Course: Therapy with IVIG instituted recently and patient has received one infusion so far. She is being evaluated by to exclude lung involvement. Points of Emphasis: DM is a systemic autoimmune disease that affects muscle, lungs and skin to varying extents in different patients. Like many other systemic autoimmune diseases, DM patients frequently have specific autoantibodies which are strongly associated with distinct clinical phenotypes, making autoantibodies useful for disease diagnosis and prognosis. Autoantibodies which recognize Mi-2 are associated with a more severe cutaneous form of DM which responds favorably to therapy while antibodies against the aminoacyl tRNA synthetases are associated with a clinical phenotype termed the “antisynthetase syndrome”, consisting of myopathy, fever, ILD, Raynaud’s phenomenon, non-erosive arthritis and mechanics hands. Autoantibodies against the interferon (IFN)-inducible antigen MDA5, have recently been described in 10–20% of Japanese DM patients and since then, in non-Japanese, non- Asian patients in other countries. Anti-MDA5 antibody-positive patients have predominantly amyopathic DM and a high risk for interstitial lung disease (ILD), including rapidly progressive ILD which was frequently fatal. Histology of MDA-5 DM may be similar to other forms of DM as in the second biopsy but other findings have included leukocytoclastic vasculitis which not characteristically seen

68 in DM. The ulcerations of the skin are also findings that may help in differentiating this more aggressive form of DM that dermatologists should now recognize. Some other important points regarding this variant include:

• Individuals with MDA5 autoantibodies typically present with a symmetric inflammatory polyarthropathy, which was frequently clinically indistinguishable from rheumatoid arthritis. A majority of MDA5 antibody-positive patients had a clinical myopathy, and ILD, when present, typically resolved with treatment.

• A majority of MDA5 antibody-positive patients exhibited three or more clinical features of the antisynthetase syndrome, with many demonstrating all six clinical features (fever, non-erosive arthritis, myopathy, Raynaud’s phenomenon, mechanics hands and ILD).

• Jo-1 antibodies, which are strongly associated with the antisynthetase syndrome, were not detected in any MDA5 autoantibody-positive patients. However, Ro52 antibodies, which are frequently found in anti-Jo-1 positive patients, were detected in 3/11 (27%) of anti-MDA5 autoantibody-positive patients.

• MDA5 antibody titers did not vary significantly over time, nor did they track with clinical course.

References:

Hall, J.C., Casciola-Rosen, L., Samedy L., et. al. Anti-MDA5-associated dermatomyositis: expanding the clinical spectrum. Arthritis Care Res (Hoboken). 2013 Aug; 65(8): 1307–1315.

69

Case No: 20

Cutaneous Collagenous Vasculopathy

Care provider: Ravi R. Patel, MD Travis Vandergriff, MD Dallas, Texas

History: A 53-year-old Caucasian male presents due to a 5-year history of an asymptomatic, progressively growing, erythematous eruption on his bilateral upper/lower extremities and abdomen. He denies fevers, chills, joint pain, or previous trauma. He had not tried any prior treatment. He has a history of Type 2 diabetes and hypertension. Current medications were metformin and amlodipine

Physical Examination: Red-brown, finely reticulated, blanchable macules coalescing into patches. No lymphadenopathy, palpable purpura, ocular, oral, or genital involvement upon further examination.

Laboratory Data: None

Histopathology: There was an attenuated epidermis with flattening of rete ridges and basket weave orthokeratosis. In the papillary dermis, there are conspicuous ectatic blood vessels with minimal inflammation. On higher power, blood vessel walls demonstrated thickened eosinophilic cuticles. There was no evidence of vasculitis. A PAS stain highlighted thickened blood vessel walls. Immunohistochemistry demonstrated to antibodies to Collagen IV in thickened blood vessel walls.

Diagnosis: Cutaneous Collagenous Vasculopathy

Clinical Course: None stated

Points of Emphasis: Cutaneous collagenous vasculopathy was first described in 2000 by Salama and Rosenthal. It is an exceedingly rare condition with only 45 reported cases in the literature although it is likely underrecognized and underreported as it has a similar clinical presentation to generalized essential telangiectasia. It is classically found in middle-aged to older individuals although there is an age range from 16 to 92 years. There is a male predominance and post patients identified thus far are Caucasian. It presents as an eruption of asymptomatic, blanching telangiectatic macules and patches that usually begins on the lower extremities before spreading to the trunk and upper extremities. The etiology is unknown although most patients have underlying comorbid conditions and are taking at least one medication. Histologic findings are exactly as seen in this case with dilated capillaries and post- capillary venules with deposition of eosinophilic hyaline material in vessel walls. The vascular deposits are derived from basement membrane and are PAS+ and diastase resistant and react to antibodies against Collagen type IV via immunohistochemistry. 70

The differential diagnosis of Cutaneous Collagenous Vasculopathy includes, pigmented purpuric dermatitis, purpuric mycosis fungoides, generalized essential telangiectasia, hereditary hemorrhagic telangiectasia, unilateral nevoid telangiectasia and scurvy. A skin biopsy can distinguish among these readily in most cases. There is no consensus regarding treatment although there are some case reports of successful treatment with lasers. Our patient started on pentoxifylline with no response thus far and because of the extent of the eruption, the cost of cost of laser therapy would be prohibitive. References 1. Burdick LM, Losher S, Somach SC, Billings SD. Cutaneous collagenous vasculopathy: a rare cutaneous microangiopathy. J Cutan Pathol 2012; 39: 741– 746.

71

Case No: 21a

ATYPICAL VASCULAR LESION

Care provider: Jenna Beasley, MD Yasmine Humeda, MS3 Pensacola, Florida

History: A 53 year-old female presented in June 2018 for asymptomatic lesions on her left breast of two years duration. Per the patient, prior evaluation by a dermatologist included a skin biopsy which revealed a “benign process” but records were not available. Her medical history is notable for an invasive ductal carcinoma of the left breast diagnosed in 2015 and treated with left lumpectomy, adjuvant radiation therapy and tamoxifen. A sentinel lymph node biopsy was negative.

Physical Examination: Examination reveals 5 purple non-scaling firm papules and thin plaques involving the left breast. Left axillary basin without lymphadenopathy.

Histopathology: 8-mm punch biopsy was performed of the left breast and reveals a vascular proliferation within the superficial to mid dermis comprised of thin-walled, ectatic vessels with focal hobnail endothelial cells. CD31, CD34 & ERG highlight the endothelial cells. MYC is negative.

Clinical Course: The patient was referred to a surgeon specializing in breast oncology. Treatment options discussed included en bloc excision versus mastectomy. Given the extent, the patient elected to proceed with mastectomy which achieved clear histologic margins. She remains without clinical recurrence at 1 year with close follow-up.

Diagnosis: Atypical Vascular Lesion

Points of Emphasis: The term “atypical vascular lesion” (AVL) was initially coined in 1994 and represents vascular proliferations that develop within a field of prior radiation (XRT), usually in the setting of prior breast cancer. While considered benign in nature, AVLs fall within a continuum and are a potential precursor to well-differentiated angiosarcoma. Cutaneous lesions typically develop within 3 to 6 years and may clinically present with erythema, telangiectasia, purpuric papules, nodules or plaques. The pathogenesis is not clearly understood but radiation is the predisposing factor.

The histopathological findings exhibit relatively well-circumscribed dilated vascular spaces within the papillary dermis lined by a single layer of endothelial cells without overt atypia. There are two histopathologic subtypes: lymphatic-type (LT) and vascular- type (VT), with the vascular type having higher risk for progression to angiosarcoma. The proliferation is positive for vascular markers (CD31, CD34, ERG); additionally, the lymphatic immunophenotype is also D240 positive. Histological findings distinguishing angiosarcomas from AVLs include intercommunicating vessels, endothelial multilayering, prominent nucleoli, mitoses, hemorrhage, destruction of the adnexa, and

72 extension into subcutaneous tissues. Features more common to AVLs include chronic inflammation, circumscription, and stromal projections into the lumen. AVLs and angiosarcoma both stain for typical vascular &/or lymphatic markers. Most radiation- associated angiosarcomas show strong MYC expression which is negative or focal in AVLs.

Standard treatment recommendations include complete excision and mastectomy. However, there are no official guidelines for surgical treatment. Close follow-up is imperative.

This case represents a classic clinicopathologic presentation of a post-radiation AVL affecting the breast and highlights the importance of appropriate biopsy, clear communication of medical history to the pathologist, definitive surgical margins and close clinical follow-up.

References: 1. Mandrell J, Mehta S, McClure S. Atypical vascular lesion of the breast. J Am Acad Dermatol 2010: 337-340. doi:10.1016/j.jaad.2009.08.017. 2. Jayalakshmy P, Sivaram A, Augustine J, Bindu P. Postmastectomy-postirradiation atypical vascular lesion of the skin: Report of 2 Cases. Case Reports in Pathology, vol. 2012, Article ID 710318, 4 pages, 2012. 3. Mattoch IW, Robbins JB, et al. Post-radiatherapy vascular proliferations in mammary skin: a clinicopathologic study of 11 cases. J Am Acad Dermatol. 2007: 126-33.

73

Case No: 21b

Organizing Thrombus Presenting as a Painful Skin Tumor

Care provider: Philip R. Cohen, MD Christof P. Erickson, MD Antoanella Calame, MD National City, California La Jolla, California

History: A 53-year-old woman presented with the acute onset of a painful lesion on her lateral left foot 2 weeks earlier. Neither prior injury nor tumor nor inflammatory skin lesion had occurred at the site.

Physical Examination: A tender, when touched, blue-black 5 x 5 mm dermal nodule within a vessel (outlined by purple ink) was noted on the lateral aspect of her left foot. There were also several varicose veins (Figs 1-2). Gross examination of the biopsy specimen revealed a black nodule in the dermis (Fig 3).

Histopathology: There was a thin-walled vascular space surrounded by a few leukocytes in the dermis. There was clotted blood showing evidence of organization within the vessel; also, there was an ingrowth of fibroblasts and capillaries (Figs 4-6).

Clinical Course: The tenderness associated with her painful tumor completely resolved after its removal during the biopsy. There has been no recurrence of the symptoms or the lesion.

Diagnosis: Organizing thrombus presenting as a painful skin tumor.

Points of Emphasis: Epstein et al originally described cutaneous organizing thrombus as a ‘capillary aneurysm of the skin’ in 1956 and 1965; Weiner subsequently described three additional patients in 1966 [1]. The lesion may present as an acutely painful tumor of the skin, clinically mimic a melanoma and microscopically demonstrated enormously dilated capillaries with thrombus formation in the upper dermis; rapid expansion of the vessel by the blood clot may result in the acute onset of pain [1]. Prior acronyms to recall the first letter of each painful skin tumors, which appear as dermal or subcutaneous nodules, include “GLENDA” and “ENGLAND” and “LEND AN EGG” [2]. “CALM HOG FLED PEN AND GETS BACK” is a new acronym—inspired by a book that many children have read (Charlotte’s Web)—that can be used to recall the list of 25 painful skin tumors: chondrodermatitis nodularis helicis, angioendotheliomatosis, leiomyoma, metastases, hidradenoma, osteoma cutis, glomus tumor, fibromyxoma, leiomyosarcoma, eccrine angiomatous hamartoma, Dercum’s disease, peizogenic pedal papule, eccrine spiradenoma neurilemmoma, angiolipoma, neuroma, dermatofibroma, granular cell tumor, endometriosis, thrombus, scar, blue rubber bleb nevus, angioma, calcinosis cutis, and keloid [3,4].

74

References: 1.Weiner MA. Capillary aneurysms of the skin. Arch Dermatol. 1966;93:670-673. 2.Naversen DN, Trask DM, Watson FH, Burket JM. Painful tumors of the skin: “LEND AN EGG”. J Am Acad Dermatol. 1993;28:298-300. 3.White EB. Charlotte’s Web. New York: Harper & Brothers, 1952. 4.Cohen PR, Erickson CP, Calame A. Painful tumors of the skin: “CALM HOG FLED PEN AND GETS BACK”. Clin Cosmet Invest Dermatol. 2019;12:1-10.

75

Case No: 22a

Angiolymphoid Hyperplasia with Eosinophilia

Care provider: Ardenne Martin Kelly Vogel, MD Jason L. Smith, MD Rome, Georgia

History: 76 y/o Caucasian male presented with multiple lesions on his head, trunk and extremities that had been present for several months. Patient reported no associated bleeding or discomfort.

Significant PMHx: Notable for prior history of basal cell carcinoma, squamous cell carcinoma, actinic keratosis, extensive exposure to solar radiation, and Bateman’s purpura. Negative family history of skin cancer.

Physical Exam: General: Well appearing, well developed and in no apparent distress Skin: Scalp : Smooth pink papule on right posterior scalp laterally – shave biopsy obtained. Trunk/Exts: multiple light brown scaling papules consistent with seborrheic keratoses Laboratory Data: None.

Histopathology: Skin biopsy of right posterior scalp. Sections demonstrate a vascular proliferation with background fibrosis and associated chronic inflammation with eosinophils. The vascular proliferation demonstrates a nodular appearance and is comprised of small to medium size vessels with a few displaying thick muscular walls. Scattered hobnailing and vacuolization of the endothelial cells is also observed. The inflammation is most pronounced surrounding the superficial vessels and becomes more sparse deeper in the dermis. The inflammatory infiltrate is comprised predominantly of lymphocytes with some scattered eosinophils. There is no evidence of malignancy.

Clinical Course: Tangential shave biopsy of skin in office (0.6 x 0.5 cm)

Diagnosis: Angiolymphoid Hyperplasia with Eosinophilia (AHLE)

Points of Emphasis: Angiolymphoid Hyperplasia with Eosinophilia typically present as single to multiple papules to nodules or varying colors, often affecting the head and neck. These benign angiomatous lesions occur in young to middle-age adults, often with a history of minor trauma or arteriovenous shunts suggesting a reactive process. Histology often demonstrates significant mural damage to or rupture of intralesional vessels. Subcutaneous nodules are characterized by invasive epithelioid endothelial cells obscuring vascular proliferation. On histology, early AHLE lesions demonstrate

76 proliferation of blood vessels containing cuboidal endothelial cells with cytoplasmic vacuoles, known as “hobnailing” of the endothelial cells. Characteristic eosinophilic and lymphocytic infiltration predominates later with development of lymphoid follicles. Background fibrosis is also common in AHLE. Most cases follow a benign course; however, lesions may co-exist with other histologically overlapping disorders, both reactive and neoplastic. Rare reports of TEK gene mutation, which codes for endothelial cell TKR Tie-2, exist. AHLE may be distinguished from Kimura diseases by large subcutaneous mass to the periauricular or submandibular region and with lymphadenopathy and increased serum levels of IgE. Angiosarcoma is also a common differential diagnosis that can be distinguished from AHLE histologically by anastomosis of vascular channels as well as mitotic activity.

References: 1. Guo, Ruifeng, and Alde Carlo P. Gavino. "Angiolymphoid hyperplasia with eosinophilia." Archives of Pathology and Laboratory Medicine 139.5 (2015): 683- 686. 2. Gonzalez-Cuyar, Luis F et al. “Angiolymphoid hyperplasia with eosinophilia developing in a patient with history of peripheral T-cell lymphoma: evidence for multicentric T-cell lymphoproliferative process.” Diagnostic pathology vol. 3 22. 29 May. 2008, doi:10.1186/1746-1596-3-22 3. Googe PB, Harris NL, Mihm MC Jr. Kimura’s disease and angiolymphoid hyperplasia with eosinophilia: two distinct histopathological entities. J Cutan Pathol. 1987;14(5):263–271.

77

Case No: 22b

ANGIOLYMPHOID HYPERPLASIA WITH EOSINOPHILIA

Care provider: Terry Henges, PA Clay J. Cockerell, MD Chapel Hill, North Carolina Dallas, Texas

History: A 37 year old African American male presented with a 6 month history of burning and itching of the left ear. He had no and he had been taking no new medications. He had been treating the lesions with topical Calamine Lotion and rubbing alcohol. He had been evaluated in an emergency clinic where he was diagnosed with “popping up keloids”.

Physical Examination: The left ear edematous on which were multiple nodules and a 1cm multilobulated pedunculated nodule. Lesions were present on the auricle, lobe and post auricular areas.

Laboratory Data: No abnormalities

Histopathology: There was a proliferation of thick-walled blood vessels in the dermis with prominent, protuberant endothelial cells. There was a dense nodular infiltrate of lymphocytes and eosinophils in the stroma.

Clinical Course: Therapy with topical Timolol 0.5% BID was instituted after the diagnosis was made with minimal benefit. This was changed to oral Proprandol 40mg po QD recently but no changes have been observed so far.

Diagnosis: Angiolymphoid Hyperplasia with Eosinophilia

Points of Emphasis: Angiolymphoid hyperplasia with eosinophilia (ALHE) most commonly presents in young to middle-aged adults as single or multiple, nondescript, skin to plum-colored papules or nodules ranging in size from a few to several centimeters. This entity is a dermal and/or subcutaneous process. Although the head and neck region is most commonly involved, the trunk, extremities, hands, penis, oral mucosa, and colon may rarely be affected. There appears to be a predilection for women, although some have reported a male preponderance.

Central to the histology of ALHE/EH is the proliferation of blood vessels of varying sizes lined by plump endothelial cells. These histiocytoid endothelial cells are enlarged with abundant eosinophilic or clear cytoplasm and large vesicular nuclei. The cells are mostly cuboidal with occasional “hobnailing”, which is related to the presence of cytoplasmic vacuoles in these cells, causing cytoplasmic protrusion into lumina. The endothelial and inflammatory cells in ALHE are typically bland. Mitoses are rare, if at all

78 present. A case that contained multinucleated giant cells has been reported but the significance of this finding is not known.

The accompanying inflammatory infiltrate is thought to play a role in the pathogenesis of the vascular proliferation. The inflammatory cells generate a proliferative stimulus to which the endothelial cells respond and cause vascular proliferation. Other possible contributing factors are, arteriovenous shunting, local trauma, and elevated serum estrogen levels.

References: 1. Ruifeng Guo and Alde Carlo P. Gavino (2015) Angiolymphoid Hyperplasia with Eosinophilia. Archives of Pathology & Laboratory Medicine: May 2015, Vol. 139, No. 5, pp. 683-686. 2. Balakrishna J. Angiolymphoid hyperplasia with eosinophilia. PathologyOutlines.com website. http://www.pathologyoutlines.com/topic/lymphnodesangiolymphoidhyperplasiawitheosin ophili.html. Accessed October 14th, 2019.

79

Case No: 23

Heparin-Induced Bullous Hemorrhagic Dermatosis

Care provider: Margaret E. Brown, MD Frank Jing, MS Huma A. Siddiqui, MD Jennifer Krejci-Manwaring, MD San Antonio, Texas

History: A 66-year-old female with a history of pulmonary sarcoidosis on chronic methotrexate and prednisone, diabetes, hypertension and Grave’s disease was admitted to the hospital for hypoxic respiratory failure. Upon admission, she was empirically initiated on intravenous antibiotics and intravenous heparin, given concern for both infection and pulmonary embolism. Dermatology was consulted for numerous painless “blood blisters” that erupted on the dorsal hands within 24 hours of admission.

Physical Exam: Physical exam revealed numerous firm, hemorrhagic papules on the dorsal hands.

Laboratory Data: Laboratory work-up revealed a slightly elevated white blood cell count (11,800/ µL), a normal, stable platelet count (231,000/µL) and a normal INR. Heparin-platelet factor 4 antibody testing was negative. Blood cultures were sterile.

Histopathology: Punch biopsy of one of the hemorrhagic papules revealed a subcorneal hemorrhagic vesicle without underlying vasculitis, vasculopathy, inflammation or viral changes. Tissue culture was sterile.

Clinical Course: After chest imaging ruled out pulmonary embolism, anticoagulation was discontinued. Her respiratory status improved on antibiotics, and her skin lesions resolved completely within two weeks.

Diagnosis: Heparin-induced bullous hemorrhagic dermatosis

Points of Emphasis: Bullous hemorrhagic dermatosis (BHD) is an under-recognized reaction to various anticoagulants. BHD presents with painless, non-inflammatory hemorrhagic vesicles and bullae occurring at sites distant from anticoagulant administration. BHD usually presents within one week of administration of an anticoagulant, but delayed onsets occurring months later have been reported.1 BHD is observed most commonly with enoxaparin use, but has also been described in association with unfractionated heparin and warfarin.1 Histologically, BHD is characterized by the presence of intraepidermal hemorrhagic bullae without thrombotic, inflammatory, or vasculitic changes. The pathogenesis of BHD remains poorly understood. Treatment of BHD should be considered in the clinical context. Discontinuation of anticoagulation, if possible, may prevent morbidity in some cases.2 If necessary to continue anticoagulation, changing the drug or decreasing the dose are

80 reasonable options. Skin lesions may resolve even if the anticoagulation is continued at the same dose.3, 4

References:

1. Russo A, Curtis S, Balbuena-Merle R, Wadia R, Wong E, Chao HH. Bullous hemorrhagic dermatosis is an under-recognized side effect of full dose low- molecular weight heparin: a case report and review of the literature. Experimental Hematology & Oncology. 2018;7(1). 2. Choudhry S, Fishman PM, Hernandez C. Heparin-induced bullous hemorrhagic dermatosis. Cutis. 2013;91(2):93-98. 3. Maldonado Cid P, Moreno Alonso de Celada R, Herranz Pinto P, Noguera Morel L, Feltes Ochoa R, Beato Merino MJ, Collantes Bellido E, López Rodríguez M, Casado Jiménez M. Bullous hemorrhagic dermatosis at sites distant from subcutaneous injections of heparin: a report of 5 cases. J Am Acad Dermatol. 2012 Nov;67(5):e220-2. 4. Snow SC, Pearson DR, Fathi R, Alkousakis T, Winslow CY, Golitz L. Heparin- induced haemorrhagic bullous dermatosis. Clin Exp Dermatol. 2018 Jun;43(4):393- 398.

81

Case No: 24

Severe Esophageal involvement in Cicatricial Pemphigoid

Care provider: Chelsea Harper DO Jean Elizze Charles DO Leah Persad DO Stephen E. Weis DO Fort Worth, Texas

History: Patient is an 89-year-old male who presented as a referral from wound care for evaluation of a back ulceration that was non-responsive to wound treatment. Upon further questioning, patient had a one-year history of scattered painful, non-healing, blistering ulcers. The initial lesion began on his left hand with subsequent lesions of mouth, right arm, back, groin, perianus, scrotum and penis. Lesions progressed from bullae into painful, draining and non-healing ulcers. Due to odynophagia, he lost more than 60lbs over six months. He saw multiple clinicians with an inconclusive biopsy. Parenteral steroids provided transient improvement.

Physical Examination: Thin, chronically ill appearing male with 8cm tender, irregular, bleeding ulcerated plaque on lower back, 6cm ulcer on proximal dorsal forearm, 1cm ulcer on posterior scalp, large irregular ulcer on hand (Figure 1 and 2). Erosions to soft palate, buccal membranes and lip were noted alongside large erosion on ventral penis extending to scrotum with fibrosis and fusion of penis to scrotum. Perianal ulcer noted with slough and stricture formation on the inferior aspect of anus. Bleeding from bilateral nares was noted with ulcer to anterior septum. No intact vesicles or bullae noted on examination.

Laboratory Data: Abnormal laboratory resulted included albumin 3.1 g/dL, (3.6-5.1), HCT 34.1%, (38.5-50), Hgb 11.2 g/dL, (13.2-17.1), total protein 5.7 g/dL, (6.1-8.1), Creatinine 1.19 mg/dL, (0.70-1.11)

Histopathology: A punch biopsy taken from the back revealed a focal subepidermal blister overlying dermal fibrosis and a superficial, mostly perivascular lymphoplasmacytic infiltrate (Figure 3a and 3b). Direct immunofluorescence studies demonstrated a smooth linear band of IgG and IgA at the dermoepidermal junction.

Clinical Course: Patient was treated with prednisone, triamcinolone 0.1% dental paste, topical betamethasone dipropionate 0.05% ointment and nutrition counseling. Studies were collected for histopathology, DIF and IDIF. A diagnosis of cicatricial pemphigoid was made based on biopsy and clinical exam findings. Inhalational budesonide 180mcg, dapsone 50mg daily and triamcinolone 1% ointment were initiated. He received inpatient care and had percutaneous endoscopic gastrostomy tube where EGD during hospital stay revealed no esophageal involvement. He had resolution of nasal ulcers and partial improvement in ulcerations. Mycophenolate and dapsone 50mg

82 were added. The perianal, anal canal and soft palate ulcerations persisted despite systemic, intralesional and topical steroids. He developed dysphagia due to food catching in his throat. A repeat EGD was performed 4 months after diagnosis, which revealed severe esophageal strictures. It was recommended against dilatation due to safety concerns about the severity and length of strictures. As a result of strictures, ongoing disability and pain, and inability to consume nutrition orally, he chose hospice care. Patient passed away 6 months after diagnosis due to his severe disease.

Diagnosis: Cicatricial Pemphigoid, IgA variant, with mucosal and non-mucosal variant without ocular involvement

Points of Emphasis: Cicatricial pemphigoid (CP) is a rare, chronic, debilitating sub- epidermal blistering disease that routinely affects the ocular, oral mucosa and rarely involves the esophagus. CP diagnosis requires a high index of suspicion and can be differentiated by analyzing the clinical presentation, histology, immunofluorescence and salt split-skin indirect immunofluorescence. The histology of cicatricial pemphigoid is relatively nonspecific. Commonly seen is a subepidermal blister that overlies a perivascular, mixed inflammatory cell infiltrate in the papillary and superficial reticular dermis. In more chronic lesions, there is marked dermal fibrosis. Direct immunofluorescent studies of perilesional skin often reveal a linear band of IgG and C3, and occasionally IgA. While the direct immunofluorescent studies are more specific than the light microscopy findings, there are still occasional false negative results. Multiple and repeated biopsies for immunofluorescence may improve the sensitivity in making the diagnosis of cicatricial pemphigoid. Erosions of the esophagus can present with dysphagia, odynophagia or be asymptomatic until chronic disease leads to strictures and dysphagia. The timeframe and severity of developing esophageal involvement is variable. When there is involvement of the esophagus, fibrosis can lead to extensive and debilitating strictures. For those with advanced disease, systemic treatments such as corticosteroids, dapsone, cyclophosphamide, mycophenolate mofetil or intravenous immunoglobulin have been used with varying degrees of success. Dapsone is reported to be an effective treatment for CP IgA variant. Our patient, however, had inadequate response to dapsone and mycophenolate mofetil, and other systemic treatments. Intervention with balloon dilatation and immunosuppression can be effective at halting progression. However, clinical remission is necessary before attempting nonpharmacological treatments as mucosal trauma has been shown to propagate disease in certain cases. We present a case report on a gentleman who initially presented with a several months history of undiagnosed cicatricial pemphigoid affecting oral, nasopharyngeal, anogenital and skin with subsequent fibrosis. Secondary to severe oral ulcers and odynophagia, involvement of his esophagus wasn’t diagnosed until late in his course. His course was subsequently fatal secondary to complications of malnutrition and severe esophageal strictures resulting from his disease. This case is useful to raise awareness of this condition. The outcome can also raise awareness of the need for early, aggressive treatment and monitoring for debilitating sequelae of cicatricial pemphigoid.

83

References: (At least two recent pertinent published references) 1. Yuan, H., Pan, M. Endoscopic characteristics of oesophagus involvement in mucous membrane pemphigoid. British Journal of Dermatology. 2017; 177(4):902-903 2. Labowsky, MT., Stinnet, SS., Liss,, J., Daluvoy, M., Hall RP 3rd., Shieh, C. Clinical implications of direct immunofluorescence findings in patients with ocular mucous membrane pemphigoid. American Journal of Ophthalmology. 2017; 183:48-55 3. Fleming TE, Korman NJ. Cicatricial pemphigoid. J Am Acad Dermatol. 2000 Oct;43(4):571-91; quiz 591-4. Review. 4. Shimanovich I, Nitz JM, Zillikens D. Multiple and repeated sampling increases the sensitivity of direct immunofluorescence testing for the diagnosis of mucous membrane pemphigoid. J Am Acad Dermatol. 2017 Oct;77(4):700-705.

84

Case No: 25

Childhood Linear IgA Bullous Dermatosis

Care provider: Fred Ghali, MD Clay J. Cockerell, MD Dallas, Texas

History: A 2 and 1/2 year old girl presented with 4 week history of a blistering eruption of the vaginal labia, around the mouth, and the upper and lower extremities. She was uncomfortable and in discomfort demonstrating notable pruritus. There were preceding illnesses and no medications had been administered previously.

Physical Examination: Multiple sausage-shaped, annular bullae of the upper and lower extremities, as well as around the mouth and genitalia were present.

Laboratory Data: No abnormalities; normal G6PD level

Histopathology: Subepidermal blister with numerous neutrophils and scattered eosinophils. Direct IF revealed a smooth linear band of IgA only.

Diagnosis: Linear IgA bullous dermatosis of childhood

Clinical Course: The patient was initially treated with a course of prednisolone, 1 mg/kg/day, tapering over 2 weeks. Dapsone was initiated at 0.5 mg/kg/day and gradually increased to provide an effective dosage. At 1 mg/kg/day, dapsone was both well-tolerated and resulted in marked clinical improvement. Her condition remains improved after 4 weeks of dapsone and laboratory parameters remain within normal limits.

Points of Emphasis: Linear immunoglobulin A (IgA) bullous dermatosis (LABD), first described by Bowen in 1901, is an autoimmune disease occurring in both children and adults. Around two thirds of cases are induced by antibiotics, nonsteroidal anti- inflammatory agents and diuretics. Childhood LABD is synonymous with chronic bullous disease of childhood (CBDC). Benign chronic bullous dermatosis of childhood (BCBDC) is characterized by predominantly monomorphous, large, tense bullae with a predilection for the lower part of the trunk, the pelvic region, and the lower extremities. Most patients have blisters for two to three years and experience spontaneous exacerbations and remissions. References: 1. K Rajendran, Abhijit Anil Patil Linear immunoglobulin A (IgA) bullous dermatosis Sri Lanka Journal of Child Health, 2018; 47: 271-272 2. Prystowsky S; Gilliam JN. Benign Chronic Bullous Dermatosis of Childhood Linear IgA and C3 Deposition on the Basement Membrane. Arch Dermatol. 1976;112(6):837-838.

85

Case No: 26

Unusual Presentation of Pityriasis Lichenoides Chronica with Overlapping Features of Erythema Multiforme

Care provider: Sarah DeVaro, BA Stephen Squires, MD Dipti Anand, MD Atlanta, Georgia

History: A 6-year-old male presented with a mildly pruritic rash involving the trunk, extremities, head and neck. Per the patient’s parents, the lesions had been steadily increasing in number and size over the course of 5 months since the family returned from a beach vacation. Lesions were refractory to treatment with diphenhydramine and loratidine. The patient had not received vaccines for over a year. He was on no other medications or supplements. No recent history of fever, blisters, colds or respiratory symptoms. Review of systems, family history, and social history were negative.

Physical Examination: Exam revealed multiple pink-colored, nontender, slightly raised macules and patches of various sizes (0.5-1.5 cm) located over patient’s trunk, extremities, head and neck. Resolution of lesions left slightly prominent post- inflammatory hypopigmentation.

Laboratory Data: Acquired over course of three office visits. KOH negative for mites and fungus. Bacterial culture recovered aeromonas sobria. Fungal culture negative for yeast or mold after 4 weeks. CBC w diff, CMP, ANA, HSV IgM, serum Tryptase all unremarkable. Mycoplasma Pneumoniae IgG positive at 0.93U/L. Plasma Histamine high at 21nmol/L.

Histopathology: Shave biopsies taken from lesions on patient’s right medial and lateral back showed interface dermatitis with basilar vacuolar alteration, intact stratum corneum, and necrotic keratinocytes with lymphocytic exocytosis. There was mild superficial dermal hemorrhage and perivascular lymphocytic infiltrate with occasional eosinophils.

Clinical Course: First course of treatment unsuccessful with Mupirocin 2% ointment BID, Ketoconazole 2% shampoo QD and Lotrisone cream. Lesions were also refractory to second course of treatment, which consisted of terbinafine 1% cream BID, Nystatin cream BID, Elidel cream BID, CeraVe SA cream BID, and Dial antibacterial soap while bathing. On third visit, patient started on Azithromycin 200mg/5mL suspension, as Erythromycin (the preferred agent for PLC), was too expensive. Regimen was 6.4mL PO on day 1 and 3.2 mL days 2-5 every other week. There was significant improvement after 2 months on Azithromycin therapy, with complete resolution after 3 months.

Diagnosis: Pityriasis Lichenoides Chronica

86

Points of Emphasis: We present a case demonstrating overlapping clinicopathologic features of Pityriasis lichenoides chronica (PLC) and Erythema Multiforme (EM). While the clinical presentation was most consistent with PLC, the histology & serologic evidence of Mycoplasma pneumoniae infection, favored EM. The patient responded to treatment with Azithromycin, a macrolide antibiotic & derivative of Erythromycin, the latter being treatment of choice for PLEVA/PLC. Pityriasis lichenoides is a spectrum of skin disorders with unclear etiology with an acute form, pityriasis lichenoides et varioliformis acuta (PLEVA), and a more chronic form, pityriasis lichnoides chronica (PLC). Proposed etiologies include hypersensitivity reaction to infectious agents like adenovirus, parvovirus B19, EBV, HIV, Staphylococcus aureus; immunizations, medications, and possibly an immune-complex mediated vasculitis. Although Mycoplasma pneumoniae has been debated as an infectious cause of PLC, it is a known cause of Erythema Multiforme (EM), an acute hypersensitivity reaction characterized by target lesions. The overall clinical course of the patient supported the final diagnosis of PLC, yet the unusual presentation with overlapping features with EM, suggests an association between the two dermatologic disorders that may be rooted in the infectious etiology. In summary, this case lends support to the addition of Mycoplasma pneumoniae to routine workup for PLC, particularly in cases refractory to treatment. Additionally, Azithryomycin should be considered as an alternative, cost effective treatment for PLC, while keeping in mind potential side effects.

References: 1. Terraneo, Luisa, et. al. Unusual Eruptions Associated with Mycoplasma Pneumonia Respiratory Infections: Review of the Literature. Dermatology. 2015; 231: 152-157. 2. Fernandes, Neil, et. al. Pityriasis Lichenoides et varioliformis acuta: a disease spectrum. Int. Journal of Derm. 2010; 49: 257-261. 3. Geller, Lauren, et. al. Pityriasis Lichenoides in Childhood: Review of Clinical Presentation and Treatment Options. Pediatric Dermatology. 2015; 32(5): 579-592.

87

Case No: 27a

Pityriasis Lichenoides-like Variant of Mycosis Fungoides (MF)

Care provider: Sarah DeVaro, BA Dipti Anand, MD Abha Soni, DO Atlanta, Georgia Smyrna, Georgia

History: A 54-year-old female presented with a generalized eruption of hypopigmented, minimally pruritic lesions on her trunk and extremities, since 6 months.

Physical Examination: There were numerous hypopigmented macules and scaly plaques with surrounding erythema involving the patient’s thighs, trunk and arms. Clinical differential diagnoses included pityriasis rosea and pityriasis lichenoides chronica.

Histopathology: Shave biopsy of lesion from left thigh showed an interface dermatitis with basilar vacuolar alteration, scattered necrotic keratinocytes, and pigment incontinence. There was epidermal acanthosis with hyperkeratosis and parakeratosis with an accompanying perivascular lymphocytic infiltrate. Epidermotropism of enlarged atypical appearing, hyperchromatic lymphocytes, worrisome for mycosis fungoides, was also seen. T-cell gamma gene rearrangement study showed evidence of monoclonality, with the same clone observed in biopsies from two different sites.

Diagnosis: Hypopigmented variant of mycosis fungoides (MF) mimicking pityriasis lichenoides chronica (PLC)

Clinical Course: The patient was started on narrowband ultraviolet B (NBUVB) phototherapy. The skin lesions markedly improved after 6 months of treatment. The patient currently remains stable with no recurrence of lesions during the 18 months of follow-up.

Points of Emphasis: MF, the most common lymphoproliferative neoplasm of skin- homing T-lymphocytes, can be a diagnostic dilemma in its early patch stage. A well- known histologic imitator of MF is Pityriasis lichenoides, especially Pityriasis Lichenoides Chronica (PLC). PLC and its acute form, PLEVA, are skin diseases of unknown etiology, that probably represent a hypersensitivity reaction to an infectious stimulus. Pityriasis lichenoides is a benign disorder, but both PLEVA and PLC can show clonal gene rearrangements. There are reports of children and adults with documented PLEVA and PLC evolving into MF. Further confounding the distinction between PL and MF are reports of children with clinical features of PLEVA, who show diagnostic histologic features of MF. PL-like MF, more commonly occurs in males, and in children and young adults (mean

88 age of 23.4 years). Presentation of this variant of MF in an elderly female is unusual. This case exemplifies the fact that the distinction between PLC and MF can be challenging, both clinically and histologically. The patient’s clinical presentation with hypopigmented scaly lesions was most consistent with PLC. Hypopigmented variant of MF typically presents with variably sized and shaped, slightly scaly patches, involving the trunk & extremities, more common in African American patients. The histology displayed features of PLC with parakeratosis, interface vacuolar alteration and scattered necrotic keratinocytes. Epidermotropism, which can be seen in PLC, was however, significant and disproportionate to the dyskeratosis, raising the suspicion of MF. The latter diagnosis was supported by demonstration of same T-cell clone in biopsies from two different sites. The clinical course and response to phototherapy, also support diagnosis of hypopigmented PLC-like variant of MF. In summary, PL-like skin lesions can appear as a specific manifestation of mycosis fungoides, and presentation of this variant is uncommon in adults. Disproportionate epidermotropism despite presence of interface change and dyskeratosis should raise the possibility of MF. T-cell gene rearrangement study can be used as an ancillary diagnostic tool. NBUVB phototherapy is a clinically effective treatment.

References: 1. Fortson J, Schroeter AL, Esterly NB. Cutaneous T-cell Lymphoma (Parapsoriasis en plaque): An Association with Pityriasis Lichenoides et Varioliformis Acuta in Young Children. Arch Dermatol. 1990;126(11):1449-1453. 2. Zaaroura H, Sahar D, Bick T, Bergman R. Relationship Between Pityriasis Lichenoides and Mycosis Fungoides: A Clinicopathological, Immunohistochemical, and Molecular Study. Am J Dermatopathol. 2018 Jun;40(6):409-415. 3. Jang MS, Kang DY, Park JB et al. Pityriasis Lichenoides-Like Mycosis Fungoides: Clinical and Histologic Features and Response to Phototherapy. Ann Dermatol. 2016 Oct;28(5):540-547.

89

Case No: 27b Patch Stage Mycosis Fungoides

Care provider: Giuseppe Cannata, MD Clay J. Cockerell, MD Imperio, Italy Dallas, Texas

History: AN 80 year old Caucasian man presented with a longstanding pruritic eruption of the trunk and extremities for several years. The condition partially responded to treatment with topical corticosteroids but recurred when discontinued.

Physical Examination: Scattered slightly scaly patches were present of the arms and some on the trunk. There was a suggestion of superficial atrophy.

Laboratory Data: N/A

Histopathology: There was a superficial patchy band-like infiltrate of lymphocytes with single cells involving the lower portion of the epidermis. A few cells were present in small clusters in upper levels of the epidermis in several foci. Some of the cells were slightly atypical.

Diagnosis: Evolving patch stage mycosis fungoides

Clinical Course: Following the diagnosis, the patient was treated with narrowband UVB phototherapy which has caused most of the lesions to regress. Points of Emphasis: Mycosis fungoides may consist of patches, plaques, or tumors, which may have a long natural history. The median duration from the onset of skin symptoms to diagnosis is 6 years. Early in the course of mycosis fungoides the skin lesions may be nonspecific, with a nondiagnostic biopsy result, so that confusion with benign conditions is common (eg, eczema, neurodermatitis,). Obtain repeated biopsies are required in patients who have a progressive chronic dermatosis or whose condition is refractory to topical treatments. Classic mycosis fungoides is divided into 3 stages: patch (atrophic or nonatrophic), plaque, and tumor. Often, the first stage goes on for many years and is characterized by a nonspecific dermatitis, which usually consists of patches and is often found on the lower trunk and buttocks. Sometimes, these patches have a thin, wrinkled quality, often with reticulated pigmentation. In this stage, pruritus is usually minimal or absent. In many cases, the disease never progresses beyond this stage.Treatment of limited patch stage disease is conservative and can usually be accomplished successfully with therapy using high potency topical corticosteroids, local radiotherapy or ultraviolet light phototherapy with narrow band UVB or topical nitrogen mustard. Patients must be monitored to detect progression. References: Cutaneous T Cell Lymphoma. Medscape. Retrieved from: https://emedicine.medscape.com/article/2139720-overview

90

Case No: 28

Telangiectasis Eruptive Macularis Perstans Variant of Mastocytosis

Care provider: Alison Black, MD Clay J. Cockerell, MD Dallas, Texas

History: A 61 year old Caucasian woman presented with a widespread reticulated eruption that was largely asymptomatic that had been present for an uncertain length of time. She had Fitzpatrick Type 1 skin with extensive sun damage and the initial impression was telangiectasia related to that. She had applied topical triamcinolone cream without benefit.

Physical Examination: Widespread reticulated telangiectatic areas of trunk, hips and thighs. Sparing of acral surfaces and mucous membranes. Dermoscopic evaluation confirmed telangiectasia but no evidence of overlying solar keratosis.

Laboratory Data: Serum tryptase 39.4 ng/ml (normal <11.5 ng/ml)

Histopathology: Sparse mononuclear cell infiltrate surrounding superficial dilated blood vessels. Immunohistochemical stain for tryptase highlighted virtually all cells in the infiltrate.

Diagnosis: Telangiectasis eruptive macularis perstans variant of mastocytosis

Clinical Course: The patient was referred to for additional evaluation and for possible bone marrow biopsy.

Points of Emphasis: Mastocytosis is characterized by an increase in the number of mast cells in one or more organs. A mutation in the c-kit proto-oncogene results in mast-cell hyperplasia. The c-kit proto-oncogene encodes KIT, a tyrosine kinase, which is the receptor for mast-cell growth factor. Although mutations in c-kit are not found in all patients with cutaneous mastocystosis, alterations in this proto-oncogene may indicate more aggressive mastocytosis. Although TMEP is a cutaneous manifestation of mastocytosis, systemic involvement may occur. Elevated serum tryptase levels may serve as a guide for systemic involvement. Treatment of patients with TMEP depends on the presence of systemic involvement or clinical symptoms. For those patients with cutaneous lesions only, management remains symptomatic with antihistamines and the avoidance of ingestants known to stimulate mast-cell degranulation. For patients with systemic disease, tyrosine kinase (TK) inhibitors have been. Imatinib mesilate (IM) inhibits a series of receptor TK including kit, abl, bcr-abl, platelet-derived

91 growth factor receptor. New tyrosine kinase inhibitors under clinical investigation for blocking KIT are dasatinib, nilotinib, masatinib mesilate and PKC412 (midostaurine). However, it is currently not clear which patients with SM will benefit from such treatments and more studies are needed to clarify the advantage of tyrosine kinase inhibitors over IFN-α or cladribine, which are currently considered first line treatment.

References: 1. Nguyen, N. Telangiectasia macularis eruptiva perstans. Derm Online J. Retrieved from: https://escholarship.org/uc/item/0gp977bv.

2. Magliacane D., Parente, R. and Triggiani M. Current Concepts on Diagnosis and Treatment of Mastocytosis. Transl Med UniSa. 2014 Jan-Mar; 8: 65–74.

92

Case No: 29

Dermal Filler Foreign Body Reaction to Nivolumab Immunotherapy

Care provider: Diana Losak, MD Clay J. Cockerell, MD Dallas, Texas

History: A 65-year old-female patient with a history of malignant melanoma presented to our clinic July of 2019, with hardening and thickening of the skin along the sides of her face for approximately two months. This problem started 3 weeks after her first infusion of nivolumab for which she was being treated for by her oncologist for malignant melanoma which had recurred two years after her first melanoma was diagnosed.

Physical Examination: Thickening of the skin along the sides of her face for approximately two months, otherise unremarkeable.

Laboratory Data: None

Histopathology: There is granulomatous inflammation with small round lobules recognizable as a filler and most consistent with hydroxyapatite.

Diagnosis: Granulomatous Dermatitis Filler with Features of Hydroxyapatite

Clinical Course: She was in good health until March of 2017 when she presented to her previous dermatologist in Kentucky with a dark spot on her left leg. An excisional biopsy confirmed malignant melanoma, Clark’s level 4, 1.25 mm in depth without ulceration and staged T2a. Her sentinel lymph node biopsy was negative. She had a wide excision of the melanoma and did not have any complications after her surgery. in March of 2019, two years after the first malignant melanoma was excised, the patient noticed another dark area growing close to the previous surgical site. A biopsy confirmed recurrence of melanoma and another wide local excision was performed with a skin graft and a negative sentinel node biopsy. Her PET scan was negative, and she was staged at T4a N0 representing IIB with recurrence. Her oncologist recommended an immunotherapy treatment with nivolumab. Her first infusion was given April 25, 2019. Pt states that the “thickening” of her skin began along the temples about three weeks after the first infusion. After the second treatment on May 23, 2019, she noticed more hardened areas which became painful along her temples, nasolabial folds, lips, and chin. Her symptoms have worsened after each infusion. She admits to having cosmetic procedures done in 2016, one of which was a dermal filler, Bellafil. Bellafil is a bovine collagen with non-resorbable polymethylmethacrylate (PMMA) microspheres. She did not have any reported reaction to the filler after the procedure and has had no other cosmetic dermal filler since that time.

93

We biopsied two areas of the affected skin which confirmed granulomatous dermatitis filler with hydroxyapatite. The patient was seen in follow up by her oncologist who consulted her plastic surgeon and recommended continuing her infusion therapy unless the foreign body reaction worsens.

Points of Emphasis: Foreign body reactions following dermal filler can be triggered by systemic infections, drugs or immunotherapy [2–4]. One case report describes a foreign body reaction during ipilimumab for malignant melanoma [2], but to date there are no reports of comparable reactions during nivolumab.Since immunotherapy is now broadly used in cancer treatment and side effects are diverse, should be aware of possible foreign body reactions. The FDG avidity of these reactions in 18F-FDG PET- CT scans highlights a possible pitfall in treatment monitoring and can occur after different cancer treatments including surgical treatment with the use of oxidized cellulose hemostatic agents for bleeding control. Physicians interpreting scans should be aware of patients’ previous cosmetic procedures, and patients treated with immunotherapy should be informed about this potential side effect.

References: 1. Bisschop C, Bruijn MS, Stenekes MW, Diercks GF, Hospers GA: Foreign body reaction triggered by cytotoxic T lymphocyte-associated protein 4 blockade 25 years after dermal filler injection. Br J Dermatol 2016;175:1351– 1353. 2. De Boulle K, Heydenrych I: Patient factors influencing dermal filler complications: prevention, assessment, and treatment. Clin Cosmet Investig Dermatol 2015;8:205–214 3. Lemperle G, Gauthier-Hazan N, Wolters M, Eisemann-Klein M, Zimmermann U, Duffy DM: Foreign body granulomas after all injectable dermal fillers: part 1. Possible causes. Plast Reconstr Surg 2009;123:1842–1863.5 Zhou Q, West DG, Shelley-Fraser G, Medford AR: Foamy macrophage deposition in lymph nodes mimicking lung cancer recurrence diagnosed via endobronchial ultrasound-guided transbronchial needle aspiration. Respiration 2015;90:426– 429. 4. Zulfiya Syunyaeva Kathrin Kahnert Diego Kauffmann-Guerrero Rudolf Maria Huber Amanda Tufman Christen Dermal Filler Injections Mimic Tumor Activity during Immune Checkpoint Inhibition M-O: Ellansé Safety: Sinclair Pharma. https://ellanse.com/wp-content/uploads/sites/2/2017/04/1605- Ellanse%CC%81eSafety-report.pdf.

94

Case No: 30

Lambda Light Chain Multiple Myeloma and Primary Systemic Amyloidosis

Care provider: Kavina Patel, BS Zachary Gillooly, MD Margaret Brown, MD Robert T. Gilson, MD San Antonio,Texas

History: A 67-year old Hispanic female with a past medical history of diabetes mellitus and unspecified leukocytosis and anemia, presented to the dermatology clinic with asymptomatic hemorrhagic bullae on the ears, postauricular scalp, chest, oral cavity, groin, and forehead. Review of systems was negative for fever, chills, night sweats, malaise, and dyspnea on exertion. Given the concern for systemic amyloidosis, baseline labs were obtained, including complete blood count (CBC), comprehensive metabolic panel (CMP), serum and urine protein electrophoresis, and the patient was referred to Hematology-Oncology.

Physical Examination: A 2 cm red, non-blanching purpuric plaque on the left temple and a large tense hemorrhagic bulla on the groin (Figure 1 and 2). Similar lesions were observed on the face, chest, and tongue (Figure 3).

Laboratory Data: CBC showed mild leukocytosis, anemia, and thrombocytopenia. Blood urea nitrogen (BUN) and creatinine were elevated. Electrophoresis showed low kappa/lambda light chain ratio and a M-spike with significantly elevated free lambda light chains in both serum and urine.

Histopathology: A punch biopsy from the left temple was performed, which showed deposits of amorphous eosinophilic material at the tips of dermal papillae and in the papillary dermis, with hemorrhage and plasma cells present (Figure 4 and 5).

Clinical Course: The patient was seen by Hematology-Oncology, and a bone marrow biopsy was performed. The biopsy demonstrated 15-20% clonal-cell population that were lambda light chain-restricted. Eosinophilic extracellular deposits were found in adjacent soft tissue and bone marrow space. This was confirmed as amyloid with apple- green birefringence under polarized light on Congo red stain and metachromatic staining with crystal violet. The patient was ultimately diagnosed with lambda light chain multiple myeloma. Further diagnostic workup included chest computed tomography (CT) and echocardiogram. Chest CT showed early pulmonary fibrosis and coronary artery calcification. Echocardiogram showed diastolic dysfunction with moderate left ventricle thickening, concerning for cardiac amyloidosis.

The patient was treated with combination therapy of bortezomib, cyclophosphamide and dexamethasone 20 mg (CyBorD) on 21-day cycles with bortezomib on days 1, 4, 8 and 11. She had received three cycles of chemotherapy before developing diarrhea,

95 hypotension, acute on chronic heart failure, and acute renal failure requiring hospitalization. She had a number of related complications due to amyloid light chain (AL)-amyloidosis and subsequently expired 16 days after her initial hospitalization from complications of MRSA bacteremia and septic shock.

Diagnosis: Lambda light chain multiple myeloma and primary systemic amyloidosis

Points of Emphasis: Primary systemic AL amyloidosis commonly affects individuals 50 to 60 years of age. Males and females are equally affected. Macroglossia and periorbital purpura are some of the pathognomonic presentations of AL amyloidosis. The major cause of death in these patients is cardiac and renal involvement, as was seen in this patient. Kidney involvement commonly presents as nephrotic syndrome, and cardiac involvement can present as a restrictive cardiomyopathy with dyspnea. Other symptoms include edema, hepatosplenomegaly, bleeding diathesis, and carpal tunnel syndrome.1

Cutaneous involvement in AL amyloidosis most commonly includes yellowish waxy papules, nodules and plaques, but can also include purpura and petechiae.1 Bullous amyloidosis is a rare cutaneous presentation of AL amyloidosis that is usually negative for the Nikolsky sign. To our knowledge, fewer than 35 cases of bullous amyloidosis have been reported in the English language literature.2 Bullae can be located intradermally or subepidermally, and are commonly hemorrhagic but can also be translucent, tender, and tense. Trauma or friction to the skin are local precipitating factors for blister formation in bullous amyloidosis.

Bullae can become apparent at any stage of AL amyloidosis, but they generally increase in size and number over time and are most common in intertriginous areas. Bullous amyloid lesions, especially those located in intertriginous areas, can have secondary impetiginization.3 In many cases, patients who present with bullous amyloidosis will ultimately be diagnosed with multiple myeloma or another plasma cell dyscrasia. In AL amyloidosis, only 10-15% of cases are associated with underlying multiple myeloma, as was the case for our patient.4

The prognosis of cutaneous amyloidosis depends on the extent of organ involvement and response to treatment. Treatment is aimed at eliminating clonal plasma cell populations to decrease the production of light chains, thereby decreasing protein burden and amyloid progression. Historical treatment options included cytotoxic chemotherapy such as oral melphalan and dexamethasone, followed by hematopoietic stem cell transplant. More recent treatment options include bortezomib, thalidomide, pomalidomide, and lenalidomide.5 Our patient received a regimen of CyBorD, which is used for patients with extensive multiple myeloma.

Cutaneous involvement of amyloidosis can be an early clue to the diagnosis of plasma cell dyscrasia. Early diagnosis and treatment can portend better prognosis and prevent progression to renal or cardiac disease.

96

References: (1) Ventarola DJ, Schuster MW, Cohen JA, Silverstein DI. JAAD grand rounds quiz. Bullae and nodules on the legs of a 57-year-old woman. J Am Acad Dermatol. 2014;71(5):1035-7. (2) Antúnez-lay A, Jaque A, González S. Hemorrhagic bullous skin lesions. Int J Dermatol. 2017;56(2):145-147. (3) Reddy K, Hoda S, Penstein A, Wasil T, Chen S. Bullous amyloidosis complicated by cellulitis and sepsis: a case report. Arch Dermatol. 2011;147(1):126-7. (4) Chu CH, Chan JY, Hsieh SW, Tuan PK, Hsiao CH, Cheng YP. Diffuse ecchymoses and blisters on a yellowish waxy base: A case of bullous amyloidosis. J Dermatol. 2016;43(6):713-4. (5) Gonzalez-ramos J, Garrido- gutiérrez C, González-silva Y, et al. Relapsing bullous amyloidosis of the oral mucosa and acquired cutis laxa in a patient with multiple myeloma: a rare triple association. Clin Exp Dermatol. 2017;42(4):410-412.

97

Case No: 31

Lichen Planopilaris with Rupture Simulating Folliculitis Decalvans

Care provider: Giuseppe Cannata, MD Clay J. Cockerell, MD Imperio, Italy Dallas, Texas

History: A 50 year old Caucasian man presented with an inflammatory alopecia of the scalp that had been present for several months. The process began as a localized area but had become more extensive.

Physical Examination: Areas of alopecia with inflammation and crust present on the frontal and central scalp. There was scale and crust at the ostia of numerous follicles and in some areas, tufted folliculitis was noted. No lesions were present elsewhere.

Laboratory Data: N/A

Histopathology: There were a number of follicles with lymphocytes around the lower infundibula and the isthmi with fibrosis. In addition, there was rupture of follicles with suppuration in the perifollicular dermis. Extensive scar formation was present.

Diagnosis: Lichen planopilaris with secondary rupture, suppuration with diffuse scarring

Clinical Course: The patient was treated with systemic and intralesional corticosteroids. The process has persisted although the acute inflammation has abated. Points of Emphasis: Lichen planopilaris (LPP) most often affects women in young adulthood. There are 3 forms: classic LPP, frontal fibrosing alopecia, and Lassueur Graham-Little Piccardi syndrome. Signs and symptoms may include patches of hair loss, scaling and redness around hair follicles, and pain, burning, tenderness, or itching on the scalp. Tiny papules may appear around clusters of hairs. Rarely, blistering occurs. In some cases, other areas of the skin or the mucous membranes are affected. Hair loss is permanent and may be extensive so that treatment should be implemented as soon as possible. Options include steroids, isotretinoin or hydroxychloroquine. Histologically, there is targeting of the mantle zone area by lymphocytes which destroy the stem cells that lead to regeneration following hair cycling into telogen. There is characteristic perifollicular fibrosis at this site. In some cases, the amount of inflammation is prominent leading to follicular rupture with secondary suppuration that may progress beyond the targeted follicles producing a pattern similar to that seen in folliculitis decalvans which can lead to confusion in diagnosis. This form is especially aggressive and can lead to extensive permanent scarring alopecia so it must be treated aggressively.

98

References: 1. Kang H, Alzolibani AA, Otberg N, Shapiro J. Lichen planopilaris. Dermatol Ther. 2008 Jul-Aug; 21(4):249-56. https://www.ncbi.nlm.nih.gov/pubmed/18715294. 2. Dyall-Smith D. Lichen planopilaris. DermNet New Zealand. 2011; https://www.dermnetnz.org/topics/lichen-planopilaris/. 3. Assouly P, Reygagne P. Lichen planopilaris: update on diagnosis and treatment. Seminars in cutaneous medicine and surgery. 2009; 28:3- 10. http://www.ncbi.nlm.nih.gov/pubmed/19341936.

99

Case No: 32

Acyclovir Resistant Anogenital Herpes Simplex in an HIV patient with Pseudoepitheliomatous Hyperplasia Resembling Squamous Cell Carcinoma

Care provider: Jean Elizze Charles, DO Stephen E. Weis, DO Gregory A. Hosler, MD, PhD Fort Worth, Texas

History: Patient is a 59-year-old African American female who presented in 2013 with complaints of vaginal discharge and erosions for several months associated with severe pain and itching that interfered with sleep. Patient has a complicated history, which includes diagnosis of HIV in 1990, history of substance abuse, and high-risk cervical human papillomavirus (HPV). Patient had recently resumed her antiretroviral therapy and her CD4 count had increased to 300 cells/mm3 from 43 cells/ mm3. Patient was clinically diagnosed with herpes simplex virus (HSV) infection and was started on valacyclovir 1gm twice daily. On follow up visit, patient experienced an overall 75% improvement in discharge as well as clearance of erosions. Patient was seen in clinic from a four-year time span from 2013 to 2016 with recurring HSV lesions, discharge and severe vulvodynia that interfered with activities of daily living despite prophylactic valacyclovir, hydrocodone-acetaminophen 10/325, naproxen, nortriptyline, and topical lidocaine.

Physical Examination: Patient has multiple exophytic ulcerations with serpiginous borders of varying sizes in the perianal region (Figure 1). Patient has 2 large vulvar ulcers (Figure 2). Patient’s labia and buttocks were inflamed.

Laboratory Data: Patient had multiple cultures and PCR positive for HSV-2 from various locations of the perianus and vulva.

Histopathology: Histologic sections of skin revealed prominent pseudoepitheliomatous hyperplasia and a mixed inflammatory infiltrate in the dermis. There was no dysplasia, acantholysis, or viral cytopathic effect.

Clinical Course: In 2016, patient’s erosions and vulvodynia continued to persist despite increasing valacyclovir 1 gram from two times daily to three times daily. Specimens were again collected and revealed lesions to be acyclovir resistant. Biopsies were taken, which revealed chronic HSV infection with pseudoepitheliomatous hyperplasia changes. At one month follow up; patient’s lesions have nearly healed after saucerization removal alongside prophylactic valacyclovir and application of topical imiquimod three times daily. At her three-month follow-up, patient had not noticed any new lesions and her physical exam revealed hypopigmented patches with complete re- epithelialization of previous lesions. Patient continues to be seen today with recurring lesions, however due to prophylactic valacyclovir 1gram twice daily and topical imiquimod 5% cream daily patient’s flares

100 were less frequent. Despite acyclovir resistance she states that her ulcerations worsen each time she stops valacyclovir. Patient’s CD4 count was 928 and viral load undetectable on April 2019.

Diagnosis: Acyclovir Resistant HSV with Pseudoepitheliomatous Hyperplasia

Points of Emphasis: Atypical HSV presentations are common in immunocompromised patients. HSV lesions that persist longer than one month, particularly atypical presentations in unusual locations, are considered one of the AIDS defining illnesses. Atypical presentations include chronic ulcerations that may be large, and hypertrophic verrucous plaques. Chronic HSV are also atypical in that they can present in extragenital locations i.e. oral, endobronchial, facial, digits of hands and feet, buttocks or back. These presentations can be easily mistaken for malignancies such as squamous cell carcinomas. Our patient’s lesions had pseudoepitheliomatous hyperplasia (PEH) on pathology, a histologic pattern that can occur in a wide variety of situations where there is chronic inflammation. While PEH is a benign condition, the histopathological finding can often be misinterpreted as squamous cell carcinoma (SCC) and it is important to rule out malignancy in patients with HIV and chronic HSV lesions. Misinterpretation clinically of atypical HSV lesions in conjunction with pseudoepitheliomatous hyperplasia on pathology can result in surgical and oncological overtreatment. Overtreatment can cause significant changes in patient quality of life. It is imperative to not overcall squamous cell carcinomas, but to have a level of suspicion of HSV especially in immunocompromised patients.

References: 1. Schneider, E., et al., Revised surveillance case definitions for HIV infection among adults, adolescents, and children aged <18 months and for HIV infection and AIDS among children aged 18 months to <13 years--United States, 2008. MMWR Recomm Rep, 2008. 57(Rr-10): p. 1-12 2. Cury, K., et al., Bipolar hypertrophic herpes: an unusual presentation of acyclovir resistant herpes simplex type 2 in a HIV-infected patient. Sex Transm Dis, 2010. 37(2): p. 126-8. 3. Zayour, M. and R. Lazova, Pseudoepitheliomatous hyperplasia: a review. Am J Dermatopathol, 2011. 33(2): p. 112-22; quiz 123-6. 4. Strehl JD, Mehlhorn G, Koch MC, Harrer EG, Harrer T, Beckmann MW, et al. HIV- associated hypertrophic herpes simplex genitalis with concomitant early invasive squamous cell carcinoma mimicking advanced genital cancer: case report and literature review. International journal of gynecological pathology: official journal of the International Society of Gynecological Pathologists. 2012;31(3):286-93.

101

Case No: 33

Spongiotic Viral Exanthem

Care provider: Fred Ghali, MD, Clay J. Cockerell, MD Dallas, Texas

History: A16 year old white female in otherwise good health presents with a recurrent, pruritic eruption that has occurred approximately 6 times within the last year. The eruption typically lasts approximately 10 days and seems to be associated with illness and/or stress. Her review of systems is otherwise unremarkable.

Physical Examination: Physical examination is remarkable for an erythematous, blanchable monomorphous popular and papulovesicular eruption distributed fairly symmetrically over the medial knees, flexural wrists, and dorsal hands and feet. The lesions do not involve the shins and the eruption spares the face and most truncal areas. They are blanchable and have a characteristic halo around many lesions.

Laboratory Data: Laboratory assessment was unremarkable; CBC, metabolic panel, sed rate, ANA titers were negative. Serologic studies were negative for acute EBV or parvovirus titers.

Histopathology: There is a superficial perivascular infiltrate consisting mostly of lymphocytes with foci of spongiosis and a slight amount of parakeratosis.

Clinical Course: The eruption resolved after 7 days without clinical sequelae.

Diagnosis: Spongiotic Viral Exanthem

Points of Emphasis: Viruses cause not only direct infectious exanthems, but also parainfectious exanthems, which provoke skin alterations via interactions with the immune system. These conditions, such as the Gianotti-Crosti syndrome, do not reflect a specific pathogen but can occur in the course of many viral infections. In addition, some exanthems result from the interaction between viruses and drugs.

Determining the cause of an exanthem is based on the characteristic morphology, distribution and time course of the eruption as well as a careful assessment of infectious contacts, immunization status and aspects of the physical examination. Although not always diagnostic, the morphology and configuration of cutaneous lesions are of considerable importance to the classification and diagnosis of viral exanthems. Serologic studies are helpful when positive but because of the wide variety of viruses that can induce these eruptions, they may not be included in panels and the causative virus is never determined as in this case.

102

References: 1. Fölster-Holst, Regina; Kreth, Hans W. Viral exanthems in childhood. Part 3: Parainfectious exanthems and those associated with virus-drug interactions Research Gate https://www.researchgate.net/publication 2. Lam, Joseph M. Characterizing Viral Exanthems Pediatr Health. 2010;4(6):623- 635. © 2010 Future Medicine Ltd.

103

Case No: 34

Disseminated Cryptococcosis

Care provider: Daniel Glade, BS Amreen Sitabkhan, MD Mary Shepherd, MD San Antonio, Texas

History: 60-year-old male presented to the emergency department with chief complaint of ulcerations for two and a half months. Patient reported he initially had two pustules on his back that ulcerated after the application of mupirocin. He subsequently developed more ulcerations, which were extremely tender, to the head, trunk, and all extremities. Review of systems was positive for fever, chills, and headache. Past medical history includes human immunodeficiency virus (CD4 16, HIV titer >2,000,000), bipolar disorder, hypertension, and polysubstance abuse. Prescribed medications include valproic acid, quetiapine, bactrim, and highly active antiretroviral therapy (dolutegravir, emtricitabine, tenofovir). Though patient reported failure to receive medications in the mail.

Physical Examination: Multiple large angulated tender ulcerations, some with purulent granulation tissue at base, to back, chest, scalp, and all four extremities. Multiple red brown papules to back, neck, and face. Punched out heme crusted circular papules to upper arms, chest, abdomen, dorsal hands, and nasal tip. No rash to palmar or plantar surfaces

Laboratory Data: Hemoglobin: 9.0 g/dl, CMP: within normal limits, CRP: 6.699 mg/L (0- 3.0 mg/L), serum cryptococcal antigen positive with titer 1:1280, CSF cryptococcal antigen negative.

Histopathology: Histiocytic and granulomatous inflammation with abundant yeast morphologically consistent with cryptococcus.

Clinical Course: Patient started on Amphotericin B and flucytosine with wound care twice weekly for dressing changes to ulcerations.

Diagnosis: Disseminated Cryptococcosis

Points of Emphasis: Cryptococcus neoformans is an encapsulated dimorphic fungus located in soil contaminated with pigeon droppings as well as decomposing fruits, vegetables, and wood. Cryptococcosis is commonly found in patients with HIV and is the second leading cause of death in Acquired Immunodeficiency Syndrome (AIDS) patients worldwide. Disseminated cryptococcosis most often results from a respiratory acquired infection that spreads to the meninges, pericardium, prostate, kidney, bone, liver, or skin. Cutaneous involvement occurs in 10-20% of cases and manifests in a

104 variety of ways including panniculitis, vasculitis, acneiform, herpetic-like, and nodular lesions. Diagnosis can be difficult and is often a diagnosis of exclusion. History and physical exam may reveal involvement of other organ systems. Sputum culture, serum and urine cryptococcal antigen titers, chest radiographs, and lumbar puncture are used for additional work-up. Histology will reveal infiltrative lymphocytes, histiocytes, and giant cells surrounding Periodic acid-Schiff (PAS) positive budding yeast. The recommended treatment is amphotericin B with either flucytosine or fluconazole.

References: 1. Beatson M, Harwood M, Reese V, Robinson-Bostom L. Primary cutaneous cryptococcosis in an elderly pigeon breeder. JAAD Case Reports. 2019;5(5):433-435. 2. Fernandez-Flores A, Saeb-Lima M, Arenas-Guzman R. Morphological Findings of Deep Cutaneous Fungal Infections. The American Journal of Dermatopathology. 2014;36(7):531-556.

105

Case No: 35

Childhood Endogenously Acquired Leishmaniasis

Care provider: Fred Ghali, MD Clay J. Cockerell, MD Dallas, Texas

History: A 4 and ½ month old white male presented with a 2 month history of a hive- like lesion on the right lateral face that had been present since the patient was 2 months of age. The parents noted that the lesion had been “raising up” and then “going back down” spontaneously. The patient lives in lives in Grayson County, Texas, a rural area, and there was no history of travel outside the area.

Physical Examination: There was a 2 cm wide urticarial plaque of the right cheek with minimal epidermal change. The lesion was asymptomatic. Although there was no urtication with rubbing, the original working diagnosis was that of solitary mastocytoma. The rest of the physical examination was unremarkable.

Laboratory Data: N/A

Histopathology: There was a diffuse dermal infiltrate of histiocytes with intracellular organisms recognizable as Leishmania.

Diagnosis: Endogenously acquired cutaneous leishmaniasis in an infant

Clinical Course: The specimen was submitted to the Centers for Disease Control in Atlanta, Georgia for identification of the causative organism which was found to represent L. mexicana. The pediatric infectious disease service was consulted and the patient was initially treated with oral fluconazole for 3 months with minimal improvement. Topical paramomycin compounded applied bid x 10 days for 3 rounds following ISDA guidelines was initiated following which the lesion improved significantly but did not totally resolve. The patient is now 12 months of age and the lesion is largely resolved but still visible. No new lesions have appeared. The current plan is observation and monthly follow-up as per recommendation of pediatric infectious disease service.

Points of Emphasis: Leishmaniasis is a vector-borne zoonotic disease caused by protozoa transmitted to human beings through the bite of the Lutzomyia sandfly. In the United States, this condition is commonly diagnosed every year in foreign travelers, immigrants, and military personnel but is rarely seen in those without foreign travel. At this time, 30 autochthonous cases of cutaneous leishmaniasis (CL) have been reported in the United States most located in south-central Texas, an area that is now known to be endemic for the disease. Recently, autochthonous cases have been reported in North Texas. In series of cases reported, all have been in adults who live in rural areas, presumably areas where patients can be exposed to the vector. This case, in an infant,

106 is the earliest known case of an endogenously acquired case of leishmaniasis in North Texas.

While most lesions have superficial erosion or crusting, this case was unusual as it clinically simulated a mastocytoma. Clinicians should be aware of this condition and consider it in the differential diagnosis of tumor-like lesions in children who reside in endemic area which now include the United States.

References:

Wright NA, Davis, LE, Aftergut, KS, et al. Cutaneous leishmaniasis in Texas: A northern spread of endemic areas. J Am Acad Dermatol 2008: 58:4, 650-652.

107